You are on page 1of 108

MRCS MESSR

A 78-year-old woman receives a course of quinolone antibiotics for a urinary tract


infection. She subsequently develops abdominal pain with foul smelling watery stools.
Which one of the following is correct regarding the diagnosis or treatment
of Clostridium difficile?
Treatment is with high-dose benzylpenicillinDiagnosis is with C. difficile toxin in
faecesDiagnosis is with C. difficile organism in faecesTreatment is with steroidsSymptoms
can be alleviated with loperamide

Explanation
Diagnosis is with C. difficile toxin in faeces
C. difficile infection should be considered in any patient who develops diarrhoea and who is
taking an antibiotic, or who has received a course of antibiotics in the past few weeks. In
the laboratory, the most reliable way of confirming the diagnosis is by the detection of the
toxin produced by C. difficile in the faeces of patients. Therefore, one or more faeces
specimens should be sent to the microbiology laboratory requesting that a specific
investigation for C. difficile is undertaken. Loperamide should be avoided as it may slow
down the rate at which toxins are cleared and may worsen C. difficile infection.
Treatment is with high-dose benzylpenicillin
Treatment is usually with oral metronidazole as a first line treatment, not benzylpenicillin.
Diagnosis is with C. difficile organism in faeces
Laboratory testing involves detection of the C. difficile toxin in the faeces, not the organism
itself.
Treatment is with steroids
Treatment with steroids is not recommended for C. difficile infection.
Symptoms can be alleviated with loperamide
Loperamide can be used as a symptomatic treatment for diarrhoea, however it should not
be used in the presence of infection.

MRCS Part A - Sep 2019 Exam


A young man, who is training for a marathon, was admitted to the Emergency Department
after passing dark brown coloured urine. The urine dipstick test for blood
was positive but no red blood cells were seen on urine sediment microscopy.

Which one of the following conditions is most likely to be associated with these
findings?

Myoglobinuria

Post-streptococcal glomerulonephritis

Renal infarction

Renal papillary necrosis

Ureteric lithiasis

Explanation
Myoglobinuria

Myoglobinuria is the presence of myoglobin in the urine, usually associated with


rhabdomyolysis or muscle destruction. Trauma, including electrical injuries and burns,
vascular problems, excessive exercise, venoms and certain drugs can destroy or damage
the muscle, releasing myoglobin into the circulation and so to the kidneys. Under ideal
situations myoglobin will be filtered and excreted with the urine, but if too much
myoglobin is released into the circulation it can overwhelm the kidneys. This can cause
casts to form (solid masses of myoglobin) which can occlude the renal filtration system,
leading to acute tubular necrosis and acute kidney injury. Patients who have undergone
severe rhabdomyolysis (eg crush injury victims) should receive high volumes of IV fluids in
an aim to prevent cast formation.

Post-streptococcal glomerulonephritis

While this can be seen in younger patients there is no preceding history of streptococcal
illness such as a sore throat in this case history.

Renal infarction
Renal infarction is rare and in a young man it would be unlikely as it is most commonly due
to thromboemboli.

Renal papillary necrosis

Papillary necrosis is most commonly caused by analgesic nephropathy (caused by


excessive ingestion of simples analgesics such as NSAIDs and paracetamol). These patients
can sometimes present with renal colic, as the necrotic tissue can obstruct ureteric
drainage.

Ureteric lithiasis

Renal stones would present with renal colic and the presence of microscopic haematuria
that would include red blood cells.

• MRCS Part A - Sep 2019 Exam


A 52-year-old woman with myasthenia gravis presents having recently experienced
urinary incontinence for the first time. She has four children and consumes caffeine daily.
She reports urgency for the last 2 years and is unable to cope with her symptoms. This is
the first time she has sought medical treatment.
Which one of the following statements is true regarding this condition?

It is usually caused by detrusor overactivity

The prevalence of the condition decreases with age until the age of 30

Bladder training does not play a role in initial management of this condition

This patient may be started on non-selective anticholinergic medication should


behavioural modifications fail

Diarrhoea is one of the common gastrointestinal (GI) side-effects of anticholinergics

Explanation
It is usually caused by detrusor overactivity

Overactive bladder (OAB) is a symptom syndrome that includes urgency with or without
urge incontinence and is usually caused by detrusor overactivity but can be due to other
causes of voiding dysfunction. OAB affects 17% of the population aged >40 years in Europe.
Symptoms include urinary urgency with or without incontinence, frequency and nocturia.
The prevalence increases with age. Conservative management such as lifestyle factors and
behavioural modifications is first line. Second line management includes use of
anticholinergics like oxybutynin, solifenacin and tolterodine. Contraindications are
mentioned above. Before starting patients on this medication, they need to be advised of
common side-effects including dry mouth, constipation, blurred vision, urinary retention
and cognitive impairment.

The prevalence of the condition decreases with age until the age of 30
Overactive bladder incidence increases with age.

Bladder training does not play a role in initial management of this condition

Behavioural modification such as:

• modifying fluid intake – not to drink past 7 pm


• avoiding stimulants like caffeine and alcohol
• bladder training – delaying micturition for increasing periods of time

have all been effective at improving symptoms.

This patient may be started on non-selective anticholinergic medication should


behavioural modifications fail

Anticholinergic medication is the mainstay of medical treatment. There are now more
selective agents such as tolterodine that reduce systemic side-effects. However,
antimuscarinics are contraindicated in the following cases:

• uncontrolled closed angle glaucoma


• myasthenia gravis
• bladder outflow obstruction
• bowel disorders.

This patient has myasthenia gravis, hence anticholinergics are contraindicated.

Diarrhoea is one of the common gastrointestinal (GI) side-effects of anticholinergics

Constipation is one of the side-effects that patients should be warned about, not diarrhoea
3453


A 25-year-old woman presents with abdominal pain and is referred as appendicitis. She
has painful urination and a positive urine dip.
Which one of the following symptoms would you also expect to find in an irritative
lower urinary tract infection?
HesitancyIncomplete voidingNocturiaOverflow incontinencePoor flow

Explanation
Nocturia
Lower urinary tract symptoms can be divided into filling (irritative) and voiding
(obstructive) symptoms. Filling symptoms consist of frequency, nocturia, urgency (with or
without urge incontinence) and suprapubic pain.
Hesitancy
Voiding symptoms consist of hesitancy, poor flow, a feeling of incomplete emptying and
postmicturition dribble. These are classed as obstructive symptoms.
Incomplete voiding
Incomplete voiding is classed as a voiding, or obstructive symptom.
Overflow incontinence
Overflow incontinence occurs when there is involuntary release of urine from an overfull
bladder. It is an obstructive symptom.
Poor flow
Poor flow is a voiding symptom. MRCS Part A - Sep 2019 Exam
You are examining a 2-week-old boy, and are unable to palpate the testis in the right side of
the scrotum. You suspect an ectopic testis.
What is the most likely site that you will find the ectopic testis lying at?
Base of the penisFemoral regionIntra-abdominalPerineumSuperficial inguinal pouch

Explanation
Superficial inguinal pouch
Testes that are not palpable in the scrotum are undescended, which can occur with an
incidence of 2–3%. Undescended testes can be retractile, ectopic or incompletely
descended. Attempts should be made to manipulate retractile testes back into the scrotum
during examination. Ectopic testes have left the normal course of descent of the testis and
approximately 80% of undescended testes fall into this category. They can be found in the
superficial inguinal pouch, (most common location), base of penis, perineum, and femoral
region.
Base of the penis
Ectopic testes are extremely rare. They can be found at the base of the penis however this
is less common than within the superficial inguinal pouch.
Femoral region
Ectopic testes have left the normal path of descent and can be found in the femoral region,
however, the superficial inguinal pouch is more common.
Intra-abdominal
Undescended testes can be found along the path of descent and testes in the abdominal
cavity are in this category.
Perineum
Testes in the perineum are considered ectopic testes, however the most likely site to find
ectopic testes is the superficial inguinal pouch.

MRCS Part A - Sep 2019 Exam


You are examining a 2-week-old boy, and are unable to palpate the testis in the right side of
the scrotum. You suspect an ectopic testis.
What is the most likely site that you will find the ectopic testis lying at?
Base of the penisFemoral regionIntra-abdominalPerineumSuperficial inguinal pouch

Explanation
Superficial inguinal pouch
Testes that are not palpable in the scrotum are undescended, which can occur with an
incidence of 2–3%. Undescended testes can be retractile, ectopic or incompletely
descended. Attempts should be made to manipulate retractile testes back into the scrotum
during examination. Ectopic testes have left the normal course of descent of the testis and
approximately 80% of undescended testes fall into this category. They can be found in the
superficial inguinal pouch, (most common location), base of penis, perineum, and femoral
region.
Base of the penis
Ectopic testes are extremely rare. They can be found at the base of the penis however this
is less common than within the superficial inguinal pouch.
Femoral region
Ectopic testes have left the normal path of descent and can be found in the femoral region,
however, the superficial inguinal pouch is more common.
Intra-abdominal
Undescended testes can be found along the path of descent and testes in the abdominal
cavity are in this category.
Perineum
Testes in the perineum are considered ectopic testes, however the most likely site to find
ectopic testes is the superficial inguinal pouch.

MRCS Part A - Sep 2019 Exam


You are examining a 2-week-old boy, and are unable to palpate the testis in the right side of
the scrotum. You suspect an ectopic testis.
What is the most likely site that you will find the ectopic testis lying at?
Base of the penisFemoral regionIntra-abdominalPerineumSuperficial inguinal pouch

Explanation
Superficial inguinal pouch
Testes that are not palpable in the scrotum are undescended, which can occur with an
incidence of 2–3%. Undescended testes can be retractile, ectopic or incompletely
descended. Attempts should be made to manipulate retractile testes back into the scrotum
during examination. Ectopic testes have left the normal course of descent of the testis and
approximately 80% of undescended testes fall into this category. They can be found in the
superficial inguinal pouch, (most common location), base of penis, perineum, and femoral
region.
Base of the penis
Ectopic testes are extremely rare. They can be found at the base of the penis however this
is less common than within the superficial inguinal pouch.
Femoral region
Ectopic testes have left the normal path of descent and can be found in the femoral region,
however, the superficial inguinal pouch is more common.
Intra-abdominal
Undescended testes can be found along the path of descent and testes in the abdominal
cavity are in this category.
Perineum
Testes in the perineum are considered ectopic testes, however the most likely site to find
ectopic testes is the superficial inguinal pouch.

MRCS Part A - Sep 2019 Exam


A 3-year-old boy is referred from his GP as his parents have noted some haematuria and
weight loss. On examination he is noted to have a visible left-sided abdominal mass.
What is the most likely cause for this?
Meckel’s diverticulumNephroblastoma
Neuroblastoma
Renal cell carcinomaTransitional cell carcinoma of the left ureter

Explanation
Nephroblastoma

Nephroblastomas (or Wilms’ tumours) occur in children <5 years and are an
undifferentiated embryonic tumour. They are the commonest intra-abdominal tumours in
children under 10 years of age and are bilateral in 10%. Presentation can be with failure to
thrive, and a visible abdominal mass. Around one-third of patients will have haematuria.
Meckel’s diverticulum
Meckel’s diverticulum will result in an acute abdomen with pain and tenderness. A visible
left-sided abdominal mass would not be seen.
Neuroblastoma

Neuroblastomas are tumours derived from neuroendocrine cells and arise from neural
crest elements of the sympathetic nervous system. They most commonly arise from the
adrenal glands, but can develop elsewhere. In most cases there will be alterations in
catecholamine levels.
Renal cell carcinoma
Renal cell carcinoma most commonly present as asymptomatic, unilateral tumours in
adults, not children.
Transitional cell carcinoma of the left ureter
Transitional cell carcinoma (TCC) would be extremely unlikely in a 3-year-old child and
instead is a disease of adults of advancing age.

MRCS Part A - Sep 2019 Exam


You are checking a patient’s blood results and notice raised prostate specific antigen (PSA)
levels. They do not complain of any symptoms of prostatism.
What can significantly elevate PSA levels?
Bedrest

Acute urinary retention

Laparoscopic cholecystectomyHydrocoele

Digital rectal examination

Explanation
Acute urinary retention

Acute urinary retention can elevate PSA levels. A mildly elevated PSA level with acute
urinary retention should be repeated before any further investigations or treatment, but a
significantly elevated PSA level should raise concerns for advanced prostate cancer.
Bedrest
There is some evidence suggesting that PSA levels decrease with bedrest although this is
not proven.
Laparoscopic cholecystectomy
Laparoscopic cholecystectomy itself has not been linked with a rise in PSA although one
study has suggested a rise in PSA level with the pneumoperitoneum created in laparoscopic
surgery, but the evidence is poor.
Hydrocoele
PSA is not affected by the presence of a hydrocele.

Digital rectal examination

Digital rectal examination can raise PSA levels but in a series of 202 patients the PSA level
only changed by 0.26 ng/ml, which has been deemed insignificant.

MRCS Part A - Sep 2019 Exam


You are treating a patient with torsion of the appendage of the testis. At exploration you
note the abnormality and excise it.
Which appendage is most commonly affected?

Epididymal appendix

Hydatid of Morgagni

Organ of Giraldés

Vas aberrans

Paradidymis

Explanation
Hydatid of Morgagni
The testicular appendix (also known as hydatid of Morgagni) is a remnant of the Müllerian
duct. It is the most common testicular appendage to tort, accounting for around 90% of
cases. Can sometimes be noted on transillumination of the testis as a black spot.

Epididymal appendix

The epididymal appendix is a remnant of the Wolffian (or mesonephric) duct and can
become torted, however it is more common for the hydatid of Morgagni to become torted.

Organ of Giraldés

This organ is a vestigial structure located on the distal portion of the spermatic cord,
however it is often not seen on imaging and is likely to be of little clinical significance.

Vas aberrans

Vas aberrans is a testicular appendage located in a fissure between the epididymis and
testis consisting of aberrant vessels. It is however of little clinical significance.

Paradidymis

The paradidymis is also known as the organ of Giraldés and does not frequently become
torted.
MRCS Part A - Sep 2019 Exam
An 18-year-old boy presents to the Emergency Department with a 2-h history of priapism.
It is extremely painful and the emergency doctor is unfamiliar with the condition and its
treatment. He contacts you, the Urology ST3 to enquire about his patient’s problem and
best treatment.
Which one of the following is true regarding priapism?

Priapism can be classified as low flow, intermediate flow or high flow only

Priapism is defined as a prolonged, unwanted erection in the absence of sexual desire or


stimulus lasting for >1 h

Priapism is associated with thalassaemia

Low-flow priapism may be treated by urgent decompression by aspirating blood from the
corpora with a 21-gauge cannula

Priapism can be a result of renal-cell carcinoma

Explanation
Low-flow priapism may be treated by urgent decompression by aspirating blood from the
corpora with a 21-gauge cannula

Management of priapism:

• Low flow: urgent decompression with aspiration of blood from the corpora. If there is
no change after 10 min, try intracavernosal injection of alpha-1-adrenergic agonist
every 5–10 min until detumescence occurs. Monitor blood pressure (BP) and pulse
during administration. If these fail after 1 h, surgical intervention may be required.
• High flow: conservative treatment recommended in most cases. Traumatic or delayed
presentations require arteriography, embolisation or ligation of fistula.

Priapism, sustained painful penile erection, is commonly iatrogenic, occurring after the
administration of intracavernosal agents used to treat erectile dysfunction. It may also
occur in sickle-cell disease and affects only the corpora cavernosa. Non-operative
treatment includes aspiration, irrigation with heparinised saline and/or administration of
intracavernosal metaraminol or phenylephrine. Surgical techniques include the formation
of a caverno-spongiosal shunt. Impotence may result, but is less likely if the condition is
effectively treated.
Priapism can be classified as low flow, intermediate flow or high flow only

Priapism can be classified into three types:

• Low flow (ischaemic): most common form due to veno-occlusion. Ischaemic priapism
> 4 h requires emergency intervention.
• High flow (non-ischaemic): due to unregulated arterial blood flow, presenting with a
semi-rigid painless erection. Usually due to trauma.
• Recurrent: most commonly seen in sickle-cell disease patients.

Priapism is defined as a prolonged, unwanted erection in the absence of sexual desire or


stimulus lasting for >1 h

Priapism is a prolonged, unwanted erection in the absence of sexual desire or stimulus


lasting for >4 h.

Priapism is associated with thalassaemia

Recurrent or stuttering priapism is associated with sickle-cell disease. It is usually high


flow but may change to low flow.

Priapism can be a result of renal-cell carcinoma

Prostate or bladder cancer that extends into the penis could cause priapism; this would be
most unusual for renal cancer.

MRCS Part A - Sep 2019 Exam


A 72-year-old woman is admitted with severe community-acquired pneumonia and goes
into acute renal failure. She is initially managed with IV fluid.
Which one of the following can be used to estimate the glomerular filtration rate
(GFR)?
ADHGlucoseInulinPara-amino hippuric acidSodium

Explanation
Inulin
Inulin (not insulin) can be used to measure the glomerular filtration rate. Inulin is freely
filtered across the glomerulus into Bowman’s capsule. It is neither reabsorbed, secreted
nor metabolised by the cells of the nephron. These serve as criteria for the measurement of
the GFR. Inulin is usually used in experimental studies. Creatinine can be used to estimate
GFR in clinical practice, creatinine is a by-product of skeletal muscle and is produced at a
relatively constant rate, any amount produced is proportional to the muscle mass. In most
clinical situations, creatinine clearance provides a reasonably accurate measure of GFR.
ADH
Anti-diuretic hormone (ADH) plays an important role in fluid homeostasis, however it
cannot be used to estimate GFR.
Glucose
Glucose in normal circumstance is not filtered by the kidney and is actively reabsorbed,
therefore cannot be used to estimate GFR.
Para-amino hippuric acid
Para-amino hippuric acid is an organic ion filtered across the glomerulus and is used to
measure renal plasma flow.
Sodium
Sodium is filtered and reabsorbed by the kidney through various mechanisms to maintain
plasma homeostasis of sodium ions and so would not be a helpful in estimating the GFR.

Contact Us

• MRCS Part A - Sep 2019 Exam


A 34-year-old man presents with a testicular lump, and describes a ‘heaviness.’ You suspect
testicular seminoma.
In testicular seminoma, which one of the following is correct?

Never present with acute testicular pain

It presents with a lump in the scrotum in 8% cases

It is radioinsensitive

It may secrete ß-HCG

The primary treatment of any testicular tumour is always radical inguinal orchidectomy

Explanation
It may secrete ß-HCG
5–10% of seminomas secrete β-HCG, which can cause gynaecomastia or mastitis.

Never present with acute testicular pain

Testicular tumours are typically painless although they can present with acute testicular
pain and this should not be forgotten when assessing an acute scrotum.
It presents with a lump in the scrotum in 8% cases
The most common presentation of any testicular tumour is a painless lump.

It is radioinsensitive

Seminomas are radiosensitive and radiotherapy is offered in select cases after radical
inguinal orchidectomy.

The primary treatment of any testicular tumour is always radical inguinal orchidectomy

This is true with one exception. If a patient presents with widespread metastatic disease
typically identified by a preoperative chest X-ray or if available on a computed tomography
(CT) scan of the chest, abdomen and pelvis. The patient should then be immediately
discussed with the oncology team and will likely undergo chemotherapy before surgery.
1404

MRCS Part A - Sep 2019 Exam


A 45-year-old man patient who is being investigated for gallstones has an ultrasound scan
(USS) of his abdomen. The radiologist notes a right-sided renal mass which ‘appears
benign’ on the report.
Which one of the following statements is true of benign renal masses?

Simple renal cysts are benign and decrease in incidence with age

Most benign renal tumours are rare, the two most clinically important are oncocytoma and
angiomyolipoma

Oncocytomas can often be distinguished radiologically from renal-cell carcinoma (RCC)

Both adult and childhood polycystic disease are not associated with cysts in the liver

Ten per cent of angiomyolipomas (AMLs) are associated with tuberous sclerosis

Explanation
Most benign renal tumours are rare, the two most clinically important are oncocytoma and
angiomyolipoma

Benign renal tumours are rare. Oncocytoma and angiomyolipomas are the most clinically
important.

Simple renal cysts are benign and decrease in incidence with age

Simple renal cysts increase in incidence with age; they are a common finding at autopsy, or
an incidental finding at USS.

Oncocytomas can often be distinguished radiologically from renal-cell carcinoma (RCC)

They often cannot be distinguished radiologically from RCCs and may even coexist with
RCCs.

Both adult and childhood polycystic disease are not associated with cysts in the liver

Both forms of polycystic kidney disease can have liver cysts. In adult polycystic kidney
disease the liver cysts have no functional significance. In childhood polycystic kidney
disease, there may be more significant liver disease ranging from bile duct proliferation
and cysts to hepatic fibrosis.

Ten per cent of angiomyolipomas (AMLs) are associated with tuberous sclerosis
About 20% of AMLs are associated with the autosomal-dominant syndrome tuberous
sclerosis which characterised by mental retardation, epilepsy, adenoma sebaceum and
other hamartomas.

• MRCS Part A - Sep 2019 Exam


You are treating a 53-year-old man who has renal calculi. He otherwise has been fairly
healthy throughout his life.
Which one of the following is associated with the greatest increased risk of renal
calculi?
HypocalcaemiaHyperparathyroidismHypothyroidism

Metanephrines

Peptic ulcer disease

Explanation
Hyperparathyroidism
Hyperparathyroidism is one of the most common causes of renal calculi.
Hyperparathyroidism causes hypercalcaemia and consequently hypercalciuria which leads
to the formation of renal calculi.
Hypocalcaemia
Hypocalcaemia does not classically result in an increased risk of renal stones. However,
there are some conditions with blood hypocalcaemia but urine hypercalciuria. This can
lead to deposits of calcium in the kidney and so stone formation, but this is rare.
Hypothyroidism
Hypothyroidism is associated with reduced glomerular filtration rate but there is no
proven association with renal calculi.

Metanephrines

Metanephrines are not associated with an increased risk of renal calculi. Urinary
metanephrines are often requested when a diagnosis of phaeochromocytoma is being
considered.
Peptic ulcer disease
Although peptic ulcer disease treated historically with milk and bicarbonate resulted in the
milk-alkali syndrome (ie associated with renal stones), the pathology of peptic ulcer
disease itself is not directly associated with renal calculi.

MRCS Part A - Sep 2019 Exam


A 52-year-old man presents to his GP with a swelling in his scrotum. This is painless, but is
becoming a nuisance due to its increasing size. On examination there is a diffuse swelling of
the scrotum, and it is not possible to distinctly palpate the testicle. When using a pen-torch
it glows red.
What is the most likely cause?
Epididymo-orchitis
Hydrocoele
Hernia
Spermatocoele
Testicular tumour

Explanation
Hydrocoele

A hydrocoele consists of fluid accumulating within the tunica vaginalis and can occur
secondary to a patent processus vaginalis in neonates, (congenital), or be idiopathic in
origin, (primary). In a hydrocoele it is often difficult to palpate the testicle separately. They
transilluminate well to produce a red glow when a pen-torch is used.
Epididymo-orchitis

Epididymo-orchitis is an inflammatory condition of the testicle and epididymis secondary


to infection. This may occur as a result of a viral infection such as mumps, or bacterial
infection due to a urinary tract infection (UTI) or a sexually transmitted disease
eg Chlamydiainfection or gonorrhoea. The epididymis is often swollen and tender.
Hernia

An indirect hernia may extend into the scrotum causing discomfort. On examination, it is
difficult to palpate above the hernia on the affected side, in contrast with a hydrocele where
one can get above it.
Spermatocoele

A spermatocoele, or epididymal cyst, is a fluid-filled cyst found within the epididymis. On


examination the testicle would be clearly palpable with a swelling noted above and behind
the affected testicle.
Testicular tumour
Testicular tumours present as painless, non-tender lumps within the testicle and are often
found on self-examination.

MRCS Part A - Sep 2019 Exam


A 27-year-old man attends the Emergency Department following an episode of
haemoptysis. He is a non-smoker and was previously well. On examination his chest and
abdomen are unremarkable, and a small 1 × 1 cm non-tender lump is found in his left
testicle that he has not previously noted. His chest X-ray shows multiple opacities in the
lung fields. He is thought to have a testicular tumour.
What is the most likely stage of this tumour?
Stage I
Stage II
Stage III
Stage IV
Stage V

Explanation
Stage IV

The Royal Marsden hospital staging system can be used for testicular tumours. It stages
them from I to IV:

Stage Details

I Tumour confined to testis


IM Rising concentrations of serum markers with no other evidence of metastases

II Abdominal node metastases


IIA ≤2cm in diameter
IIB 2-5cm in diameter
IIC >5cm in diameter

III Supradiaphragmatic nodal metastases


ABC Node stage as defined in Stage II
M Mediastinal
N Supraclavicular, cervical or axillary
O No abdominal metastases

Extralymphatic metastases
IV
Lung ≤3 metastases
L1 ≥3 metastases, all ≤2cm in diameter
≥3 metastases, one or more of which are ≤2cm in diameter
L2
L3

Stage I

Stage I would be classified as a tumour confined to the testis.


Stage II

Stage II indicates the presence of abdominal metastases.


Stage III

Stage III indicates the presence of supradiaphragmatic metastases affecting either the
mediastinal, supraclavicular, cervical or axillary lymph nodes.
Stage V
Stage V does not exist in the Royal Marsden staging of testicular tumours.

MRCS Part A - Sep 2019 Exam


A 52-year-old man presents to the Urology department with excruciating intermittent pain
in his left flank radiating to his groin. He has associated sweating, nausea, vomiting, and
urinary urgency. A diagnosis of renal stones is made.
In the management of renal stones, which one of the following is
a contraindication to percutaneous nephrolithotomy (PCNL)?
Abdominal aortic aneurysmsClotting abnormalitiesLarge-volume stonesStones in a
transplanted kidney

Patients with urinary diversion in the form of an ileal conduit

Explanation
Clotting abnormalities
Clotting abnormalities should be corrected before PCNL, this diagnosis is also a
contraindication for extracorporeal shock wave lithotripsy. In patients with an obstructing
calculus requiring emergency intervention and clotting abnormalities, a ureteric stent in
the first instance would be appropriate to decompress the kidney.
Abdominal aortic aneurysms
Abdominal aortic aneurysms are not a contraindication to PCNL but are
to extracorporeal shock wave lithotripsy due to the risk of rupture.
Large-volume stones
PCNL is the treatment of choice for large-volume stones in the kidney. It should be
considered first line for stones >20 mm.
Stones in a transplanted kidney
PCNL is a safe treatment for stones in a transplanted kidney but should be carried out in a
specialist centre.

Patients with urinary diversion in the form of an ileal conduit

PCNL is often considered in patients with kidney stones that have undergone urinary
diversion. A retrograde approach can also be considered if access to the ureters can be
gained.
1389

• MRCS Part A - Sep 2019 Exam


A 52-year-old man presents to the Urology department with excruciating intermittent pain
in his left flank radiating to his groin. He has associated sweating, nausea, vomiting, and
urinary urgency. A diagnosis of renal stones is made.
In the management of renal stones, which one of the following is
a contraindication to percutaneous nephrolithotomy (PCNL)?
Abdominal aortic aneurysmsClotting abnormalitiesLarge-volume stonesStones in a
transplanted kidney

Patients with urinary diversion in the form of an ileal conduit

Explanation
Clotting abnormalities
Clotting abnormalities should be corrected before PCNL, this diagnosis is also a
contraindication for extracorporeal shock wave lithotripsy. In patients with an obstructing
calculus requiring emergency intervention and clotting abnormalities, a ureteric stent in
the first instance would be appropriate to decompress the kidney.
Abdominal aortic aneurysms
Abdominal aortic aneurysms are not a contraindication to PCNL but are
to extracorporeal shock wave lithotripsy due to the risk of rupture.
Large-volume stones
PCNL is the treatment of choice for large-volume stones in the kidney. It should be
considered first line for stones >20 mm.
Stones in a transplanted kidney
PCNL is a safe treatment for stones in a transplanted kidney but should be carried out in a
specialist centre.

Patients with urinary diversion in the form of an ileal conduit

PCNL is often considered in patients with kidney stones that have undergone urinary
diversion. A retrograde approach can also be considered if access to the ureters can be
gained.
1389

• MRCS Part A - Sep 2019 Exam


A 75-year-old man attends the clinic with predominantly voiding lower urinary tract
symptoms (LUTS) including hesitancy, poor flow, terminal dribbling and nocturia. He is
counselled about further tests and investigations to further assess the cause of his
symptoms.
Which one of the following is not one of the next steps/first-line investigations in the
evaluation of a patient with LUTS?

Digital rectal examination (DRE)

Transperineal prostate biopsy

Prostate specific antigen (PSA)

Serum creatinine

Post-void residual (PVR) urine volume and flow rate

Explanation
Transperineal prostate biopsy

This is not one of the first-line investigations that should be performed. The patient may
subsequently go on to have a biopsy of the prostate gland, however it is an invasive
procedure and should not be done without other vital information obtained through less
invasive methods. The following investigations/tests are considered as first line in
investigation of LUTS:

• DRE
• prostate specific antigen (PSA)
• serum creatinine
• post-void residual volume (PVR)
• flow rate measurement
• pressure flow studies
• renal ultrasonography.

Digital rectal examination (DRE)

A digital rectal examination is done to detect nodules that may indicate an underlying
prostate cancer as well as to estimate the consistency and rough size of the prostate. This
must be done when assessing a patient with LUTS.
Prostate specific antigen (PSA)

PSA testing is important and a patient must be counselled on to what the results may then
lead before obtaining this blood sample. There are acceptable PSA ranges according to a
patient’s age, other factors such as prostatitis, vigorous physical activity and instrument
use can affect the PSA result.

Serum creatinine

Baseline measure of renal function as well as detection of renal failure secondary to high-
pressure urinary retention is a very important first-line investigation.

Post-void residual (PVR) urine volume and flow rate

Post-void residual urine volume and flow rate are important measurements as they give
much information on how the urinary flow is affected. They are obligatory before
undertaking any surgical treatment for benign prostatic hyperplasia. They also give a good
picture of the severity of the symptoms and how they are likely to affect the patient’s
quality of life.
152


• MRCS Part A - Sep 2019 Exam

A 25-year-old scaffolder presents to the Urology Clinic with a 3-month history of swelling
in the left hemiscrotum. On further questioning, he reveals he had an orchidopexy for
an undescended left testicle when he was an infant. Clinical examination reveals a
smooth, hard lump in the left hemiscrotum that cannot be differentiated from the testicle
itself. It does not transilluminate.
What is the optimal management of this patient?
Needle aspiration

Ultrasound scan and needle aspiration

Ultrasound scan, beta-human chorionic gonadotrophin (b-HCG) and alpha-fetoprotein


blood tests and if positive orchidectomy and/or chemotherapy

Ultrasound scan, beta-HCG and alpha-fetoprotein blood tests and Jaboulay’s repair

Ultrasound scan, beta-HCG, lactate dehydrogenase (LDH) and alpha-fetoprotein blood tests
and, if positive, a discussion in the multi-disciplinary team (MDT) meeting prior to
orchidectomy

Explanation
Ultrasound scan, beta-HCG, lactate dehydrogenase (LDH) and alpha-fetoprotein blood tests
and, if positive, a discussion in the multi-disciplinary team (MDT) meeting prior to
orchidectomy

Initial blood test markers must be measured at presentation: alpha-fetoprotein, HCG and
LDH. An ultrasound of the testes can further characterise the lump and a staging CT scan
would assess the extent of cancer spread and be done prior to or after surgery. An MDT
discussion is the gold standard and a very important part of all cancer management. A
testicular lump in a young man should always evoke suspicion for a cancer, particularly in
this patient with a positive risk factor, which is a history of undescended testicle which
increases his risk by about eight-fold. Differential diagnoses of a testicular lump include:

• hydrocele
• epididymal cyst
• testicular malignancy
• indirect inguinal hernia.
A hydrocele is the abnormal accumulation of fluid within the tunica vaginalis. Secondary
hydroceles may result from testicular injury, tumour, heart failure or obstruction of the
retroperitoneal lymphatics. The Jaboulay procedure involves excision and eversion of the
hydrocoele sac and Lord’s procedure is plication of the sac. In this age group, a secondary
hydrocoele caused by testicular malignancy is a real possibility. Non-seminomatous germ-
cell tumours are commoner in the age group 20–30 years for which the tumour markers
alpha-fetoprotein and beta-HCG are elevated.
Needle aspiration
This option is not appropriate as there is a suspicion that this might be a testicular
malignancy. Aspiration is contraindicated as it has the possibility of seeding the cancer
cells.

Ultrasound scan and needle aspiration

An ultrasound of the testicle is not an unreasonable suggestion, however other important


tests should be conducted. Aspiration is not appropriate here.

Ultrasound scan, beta-human chorionic gonadotrophin (b-HCG) and alpha-fetoprotein


blood tests and if positive orchidectomy and/or chemotherapy

A multi-disciplinary team (MDT) discussion is crucial before a decision of management of


any cancer.

Ultrasound scan, beta-HCG and alpha-fetoprotein blood tests and Jaboulay’s repair

Jaboulay’s repair is indicated for hydroceles, not in suspected testicular malignancy. An


ultrasound scan is usually obtained before surgery to assess the size and confirm the extent
of the hydrocele.

• MRCS Part A - Sep 2019 Exam


A concerned mother brings her 7-month-old infant to the Paediatric Urology Clinic. She
tells you she has noticed ‘an opening of the water pipe’, which, on examination, is
confirmed to be a hypospadias.
Which one of the following statements is correct regarding hypospadias?

Is a congenital deformity in which the opening of the urethra (meatus) is sited on the
dorsal part of the penis

It can be classified into anterior and posterior only

Associated anomalies include undescended testes, inguinal hernia, disorders of sexual


development (DSD) and hydroceles

Repair is performed after 3 years of age

Urethral diverticulum is not a known complication of surgical repair of hypospadias

Explanation
Associated anomalies include undescended testes, inguinal hernia, disorders of sexual
development (DSD) and hydroceles

Hypospadias are associated with other genitourinary anomalies such as undescended


testes, inguinal hernia, DSD and hydroceles due to it being an embryological defect.

Is a congenital deformity in which the opening of the urethra (meatus) is sited on the
dorsal part of the penis

In hypospadias, the opening of the urethral meatus in fact sites on the underside (ventral)
part of the penis.

It can be classified into anterior and posterior only

Hypospadias can be classified according to the anatomical location of the urethral meatus.
There are three types:

• anterior – most common (50–80%)


• middle
• posterior – penoscrotal, scrotal and perineal.

Repair is performed after 3 years of age


Surgery is indicated when the deformity is severe, interferes with voiding or is predicted to
interfere with sexual function. Repair is performed between 6–18 months of age.

Urethral diverticulum is not a known complication of surgical repair of hypospadias

Complications of surgical repair include urethrocutaneous fistula, urethral stricture, poor


cosmesis, urethral diverticulum, meatal stenosis, spraying of urine and voiding dysfunction.
238

• MRCS Part A - Sep 2019 Exam


An 18-year-old man presents to Accident and Emergency with acute pain passing urine,
haematuria and a dull ache over his flank. He is known to have cystinuria.
Which one of the following statements about cystinuria is correct?
Can be prevented with allopurinol therapy

Renal stones may be demonstrated by ultrasonography

May be caused by d-Penicillamine

Occur more commonly than uric acid stonesTend to precipitate in alkaline urine

Explanation
Renal stones may be demonstrated by ultrasonography

Renal stones can be detected on ultrasonography, which is particularly useful in young


patients and pregnant patients when you do not want to expose them to the radiation of
a computed tomography (CT) scan of the kidneys, ureters and bladder (KUB).
Can be prevented with allopurinol therapy
Allopurinol therapy can be used to help prevent uric acid stones but are not useful for
cystine stones.

May be caused by d-Penicillamine

d-Penicillamine can be used to help prevent cystine stones and therefore does not cause
them. It combines with cystine to form a soluble complex preventing stone formation.
Occur more commonly than uric acid stones
The incidence of uric acid stones is 8–10% with cystine stones occurring in 1–2% of
patients.
Tend to precipitate in alkaline urine
Cystine stones tend to precipitate in acidic urine. Alkalinisation with potassium citrate is a
treatment for cystinuria.
1388

• MRCS Part A - Sep 2019 Exam


A 32-year-old man presents with a 2-day history of left-sided lower abdominal and scrotal
pain. He is pyrexial at 37.8°C and visibly in pain. On examination, it is evident that the left
testicle is erythematous, hot to the touch and swollen.
Which one of the following statements is true regarding the most likely diagnosis?

It is most commonly caused by a virus

In men aged <35 years, the most common pathogen is Staphylococcus aureus

Amiodarone can potentially cause this condition

Treatment of choice is usually a 7-day course of an antibiotic

A complication of this condition commonly includes haematoma formation

Explanation
Amiodarone can potentially cause this condition

A rare, non-infectious cause of epididymitis is amiodarone, the antiarrhythmic drug which


accumulates in high concentrations within the epididymis, causing inflammation. It can be
unilateral or bilateral and resolves on discontinuation of the drug.

It is most commonly caused by a virus

Acute epididymitis is inflammation of the epididymis and is most commonly caused by


bacterial infection.

In men aged <35 years, the most common pathogen is Staphylococcus aureus

In sexually active men aged under 35 years, the most common pathogen is usually Neisseria
gonorrhoea, Chlamydia trachomatis or coliform. In older men and children, Escherichia
coli is more common.

Treatment of choice is usually a 7-day course of an antibiotic

Antibiotics is the mainstay of treatment until urine culture sensitivities are available,
however it is typically given over a 2-week period. Doxycycline or ciprofloxacin is usually
prescribed, depending on suspected aetiology.

A complication of this condition commonly includes haematoma formation

Recognised complications of epididymitis are:


• abscess formation
• infarction of the testis
• chronic pain and infection
• infertility

• MRCS Part A - Sep 2019 Exam.


A 30 year-old man attends the Emergency Department complaining of left flank pain. He is
visibly in pain despite 10 mg of intravenous (IV) morphine. His urine dip is positive for
nitrites and he is pyrexial at 38°C.
Which one of the following statements is correct about his most likely diagnosis?

The first step in management would be getting a CTKUB

It is most likely to be due to a calcium oxalate calculus

It is not necessary to send off a urine culture as the urine dip is positive for nitrites

Corticosteroids are not a known risk factor in stone formation

First line imaging would be an ultrasound of the renal tract

Explanation
It is most likely to be due to a calcium oxalate calculus

80–85% of all kidney stones are made of calcium oxalate and is therefore the most likely
composition of the kidney stone. Stone disease is becoming a growing problem in the
population. Intrinsic and extrinsic factors both play a role in predisposition. Risk factors for
stone disease include:

• diet: fluid intake, meat consumption, high uric acid levels


• drugs: corticosteroids, chemotherapeutic agents
• urinary tract infection (UTI)
• immobility
• systemic disease: gout, primary hyperparathyroidism, sarcoidosis
• family history: cystinuria
• renal anatomy
• previous bowel resection.

The first step in management would be getting a CTKUB

Always manage a patient, as in this case for example, who is likely to be septic using an
ABCDE (Airway maintenance with cervical spine protection, Breathing and
ventilation, Circulation with haemorrhage control, Disability: neurological
status, Exposure/environmental control) approach to conduct a safe assessment and
address any issues in that order.
It is not necessary to send off a urine culture as the urine dip is positive for nitrites

It is essential that the urine is cultured so ensure an midstream specimen of urine (MSU) is
always sent off. This is useful to know in terms of antimicrobial treatment and will guide
future treatment should repeated episodes occur.

Corticosteroids are not a known risk factor in stone formation

Corticosteroids are a known risk factor as they increase enteric absorption of calcium,
leading to hypercalciuria accelerating stone formation.

First line imaging would be an ultrasound of the renal tract

Renal stones are not always detected on renal tract ultrasonography (USKUB) hence a plain
abdominal radiograph is preferable, especially in this case as it is an acute presentation
with no contraindication to radiation for example, a pregnant woman. In some trusts, a
computed tomography (CT) KUB is done as first line imaging however this should be
considered on a case by case basis taking into account radiation risk.
3748

MRCS Part A - Sep 2019 Exam


A 35-year-old man has a clinical diagnosis of a testicular malignancy in a
previously maldescended testis.
From the following list of testicular malignancies which is the most likely histological
diagnosis?
Interstitial (Leydig) cell tumourLymphomaSeminomaTeratomaYolk sac tumour

Explanation
Seminoma
Seminomas are a type of germ cell tumour along with teratomas they account for 85–90%
of testicular tumours. Peak incidence of seminomas is 40–60 years. Seminomas are the
commonest testicular tumours to develop in maldescended testicles with a 10–40 times
greater risk.
Interstitial (Leydig) cell tumour
Leydig cell tumours are rare testicular tumours. They is a type of sex cord stromal tumour
that can occur in prepubertal boys but are most common in men aged 30–60 years.
Lymphoma
Testicular lymphomas are a rare extranodal presentation of non-Hodgkin’s lymphoma. It
has a peak incidence of 60–70 years and is more common than primary testicular cancer in
men over 50 years.
Teratoma
Teratomas are of germ cell origin with a peak incidence of 20–30 years, but it is not the
most common testicular tumour in a maldescended testis.
Yolk sac tumour
Yolk sac tumours occur before the age of 3 years and are the commonest testicular
malignancy in children.
1453


• MRCS Part A - Sep 2019 Exam

A 21-year-old man presents with pain in his left scrotum that has been increasing for the
past 2 days. He is systemically well and on examination his abdomen is soft and non-
tender. On palpation the left testicle is slightly tender with no palpable lumps, but he is
very tender slightly above and behind the testicle.
What is the most likely diagnosis?
Epididymo-orchitis
Hydrocoele
Inguinal hernia
Testicular tumour
Torsion

Explanation
Epididymo-orchitis

Epididymo-orchitis is an inflammatory condition of the testicle and epididymis secondary


to infection. This may occur as a result of a viral infection such as mumps, or bacterial
infection due to a urinary tract infection (UTI) or sexually transmitted disease
eg Chlamydia infection or gonorrhoea. The epididymis is often swollen and tender.
Hydrocoele

In a hydrocoele the testicle is often impalpable and it is usually painless.


Inguinal hernia

An inguinal hernia extending into the scrotum would cause a lump and it is not possible to
‘get above it’.
Testicular tumour

Testicular tumours often present as painless, palpable lumps often found on self-
examination.
Torsion
Testicular torsion is an acute surgical emergency with an exquisitely tender testicle.
2554

• MRCS Part A - Sep 2019 Exam


A 71-year-old man presents to his GP with a history of frank haematuria. He denies any
associated pain and is generally well in himself. Examination is unremarkable and his only
past medical history of note is chronic obstructive pulmonary disease (COPD) secondary to
a long smoking history.
What is the most likely cause for the haematuria?
Bladder cancer

Benign prostatic hypertrophy (BPH)

Prostate cancerRenal colic

Urinary tract infection (UTI)

Explanation
Bladder cancer
Painless haematuria in any patient above the age of 50 should be considered to be the
result of bladder cancer until proven otherwise. The main risk factor for transitional-cell
carcinoma (TCC) of the bladder in the UK at present is smoking, of which this man has a
significant history.

Benign prostatic hypertrophy (BPH)

BPH presents with lower urinary tract symptoms such as increased frequency, hesitancy
and poor flow. Haematuria is uncommon in BPH and should raise the suspicion of bladder
cancer.
Prostate cancer
Similarly, prostate cancer presents with lower urinary tract symptoms, not haematuria.
Renal colic

Renal colic usually presents with loin to groin pain with microscopic haematuria.

Urinary tract infection (UTI)

UTI may present with haematuria, however this is often microscopic and associated with
dysuria and other systemic symptoms of infection.

MRCS Part A - Sep 2019 Exam


You are discussing cases in the urology MDT. The pathologist discusses the details of some
of the cancers encountered at the meeting.
Which one of the following malignancies is most commonly multi-centric?

Testicular cancer

Renal cell carcinoma

Squamous cell carcinoma of penis

Transitional cell carcinoma

Wilms' tumour of the kidney

Explanation
Transitional cell carcinoma

Transitional cell carcinomas can occur anywhere in the urothelium and multi-centricity
and recurrence are common.

Testicular cancer

Synchronous testicular cancers are very rare.

Renal cell carcinoma

Renal cell carcinomas are sometimes bilateral or multi-centric, but not often.

Squamous cell carcinoma of penis

Generally, penile carcinomas are solitary (but infiltrative) mass lesions.

Wilms' tumour of the kidney

Wilms’ tumour usually presents as a solitary mass, though about 10% are bilateral or
multi-centric at the time of diagnosis.


• MRCS Part A - Sep 2019 Exam

Some 24 h after sustaining severe skeletal and soft tissue trauma in a road traffic accident,
a 19-year-old man becomes oliguric. The nurses ask you to review him as his urine output
is now persistently minimal.
Which one of the following would most suggest that this is pre-renal in origin, rather
than being due to acute tubular necrosis?

Glycosuria

Hyperkalaemia

Hyperuricaemia

Urinary osmolality less than 250 mmol/kg

Urinary sodium concentration less than 20 mmol/l

Explanation
Urinary sodium concentration less than 20 mmol/l

A urinary sodium concentration less than 20 mmol/l suggests that renal tubular function is
intact. Whereas if oliguria is due to acute tubular necrosis, it would be expected that
urinary sodium concentration will be over 80 mmol/l, due to leaching of sodium from the
tubules with a urine:plasma osmolality of 1:1.

Glycosuria

Trauma can cause transient glycosuria because of a stress-induced rise in blood glucose
concentration.

Hyperkalaemia

Tissue breakdown and decreased urine production can cause both hyperkalaemia and
hyperuricaemia.

Hyperuricaemia

As discussed, hyperuricaemia can occur following tissue breakdown from trauma, however
it does not provide an indication as to whether the origin of reduced renal function is intra-
renal or pre-renal.
Urinary osmolality less than 250 mmol/kg

Urine osmolality is a measure of concentration of the urine and if the patient is suffering
from pre-renal failure then the osmolality would be high due to reabsorption of water from
the kidneys under the actions of ADH.

MRCS Part A - Sep 2019 Exam


A 67-year-old man life-long smoker undergoes an urgent cystoscopy for haematuria. A
bladder tumour is found and biopsied.
What is the most common type of bladder cancer in the UK?
AdenocarcinomaBenign tumourSarcomaSquamous Cell Carcinoma (SCC)Transitional cell
carcinoma (TCC)

Explanation
Transitional cell carcinoma (TCC)
Ninety per cent of bladder cancers are TCC, 5% are SCC and the rest are adenocarcinomas.
Sarcoma is rare and benign tumours even more uncommon. TCC is an urothelial tumour of
the type that lines the entire urinary tract. Ninety-five per cent affect the bladder whilst 5%
affect the upper tract. Carcinoma of the bladder is more common in men (ratio of
men:women = 3:1) in whom it is the fourth most common cancer (after prostate, lung and
colorectal tumours). Haematuria is commonly painless although some may present with
cystitis, or ureteric colic (if the tumour obstructs ureteric flow). Superficial tumours may be
managed with trans-urethral resection of bladder tumours (TURBT) in conjunction with
intra-vesical therapy such as Bacillus Calmette–Guérin (BCG) vaccine or mitomycin. These
patients require frequent surveillance with cystoscopy. Muscle invasive tumours are
generally managed by radical cystectomy, assuming that the patient has a reasonable
performance status.
Adenocarcinoma
Less than 5% of bladder tumours are adenocarcinomas.
Benign tumour
Similar to sarcomas, benign tumours of the bladder are rare.
Sarcoma
Sarcoma is a rare form of bladder cancer.
Squamous Cell Carcinoma (SCC)
Only 5% of bladder tumours are squamous cell carcinomas. Squamous cell carcinoma is
associated with chronic bladder irritation – such as long-term catheters, bladder stones
and schistosomiasis infection.

MRCS Part A - Sep 2019 Exam


A 47-year-old man with poorly controlled diabetes and hypertension is referred to a renal
clinic due to concerns regarding his renal function. It is decided to monitor his glomerular
filtration rate (GFR) as an outpatient.
Which one of the following is used in clinical practice to estimate GFR?
Creatinine levelCreatinine clearanceInulin clearanceUrea levelsUrine output

Explanation
Creatinine clearance
The GFR varies during a lifetime. Adult levels are reached at 2 years of age and decline
linearly from 40 years due to glomerular sclerosis. In clinical practice, GFR is estimated by
measuring the creatinine clearance. A 50% increase in GFR occurs during pregnancy. GFR
increases after a protein-rich meal.
Creatinine level
Isolated creatinine levels alone cannot be used to estimate GFR, instead creatinine
clearance is used to estimate GFR.
Inulin clearance
In research laboratories, GFR can be measured by inulin clearance. Inulin is a polymer of
fructose that is not secreted, metabolised or reabsorbed by the kidney. This means that the
amount of inulin excreted in the urine (per min) equals the amount of inulin filtered at the
glomerulus (per min).
Urea levels
Urea levels provide an indication of renal function but they are not used in the calculation
of estimated GFR.
Urine output
Urine output provides a gross indication of renal function, however it is not used to
estimate GFR.
1798

• MRCS Part A - Sep 2019 Exam


A 54-year-old woman with chronic glomerulonephritis requires a renal transplant as she
has end-stage renal failure (ESRF). A suitable kidney has become available in the last 4
hours.
The renal artery and vein of the donor are most likely to be anastomosed to which
vessels of the recipient in end to side anastomosis?
Aorta and inferior vena cava
External iliac artery and vein
Femoral artery and vein
Internal iliac artery and vein
Renal artery and vein

Explanation
External iliac artery and vein

The kidney is placed extraperitoneally in the right or left iliac fossa, usually on the right
side (dictated by existing scars or previous transplants and because it is relatively
straightforward to access for biopsies and surgery in the event of post-operative
problems). Renal vessels are anastomosed (end to side) to the recipient’s external iliac
vessels. The ureter is taken down into the pelvis where it is anastomosed to the bladder
mucosa, either directly (extravesical approach) or by threading it through a submucosal
tunnel and suturing from inside the bladder through a separate incision in the bladder wall
(intravesical approach).
Aorta and inferior vena cava

This would require placement of the transplant kidney deep into the abdominal cavity
which would prevent easy future access for biopsies and post-operative complications.
Femoral artery and vein

The femoral vessels start below the level of the inguinal ligament and therefore it would
not be accessible for a transplanted kidney in the iliac fossa.
Internal iliac artery and vein

The internal iliac vessels run deep in the pelvis close to the sacrum and therefore are
suboptimal vessels to perform the anastomosis.
Renal artery and vein
The transplanted kidney is placed in the right or left iliac fossa and therefore would not be
anastomosed with the renal artery and vein.
Contact Us
• MRCS Part A - Sep 2019 Exam

A 15-year-old boy is brought into A&E by his parents who are concerned he has started
limping and complaining of pain in the lower abdomen. He also gives a history of nausea
and vomiting. On examination his abdomen is soft throughout, but his right testicle is noted
to be very tender, swollen and noted to be drawn up in the scrotum.
What is the most likely diagnosis?
Appendicitis
Diverticulitis
Epididymo-orchitis
Meckel’s diverticulumTorsion

Explanation
Torsion
When considering causes of a limping child it is important to think of causes beyond the
musculoskeletal system, such as those relating to general surgical and urological
conditions. These can include causes of an acute abdomen such as appendicitis or a
Meckel’s diverticulum. However, in this case the abdomen is soft throughout and the right
testicle is noted to be exquisitely tender and drawn up. In any boy of this age presenting
with this history and examination findings, a torted testicle must be considered. Torsion is
a urological emergency and the testis can infarct within hours if not recognised and treated.
Treatment consists of surgical exploration and if a torted testicle is seen, it should be
untwisted and then secured in place (orchidopexy) or if it is found to be not viable an
orchidectomy performed. The other testicle must also be secured as well to prevent
possible future torsion of that testicle.
Appendicitis

Appendicitis would present with abdominal pain and tenderness on examination. It can
cause a child to limp, however due to inflammation of the peritoneum in the right iliac
fossa.
Diverticulitis

Diverticulitis would be extremely unlikely in a 15-year-old boy.


Epididymo-orchitis

Epididymo-orchitis is an inflammatory condition of the testicle and epididymis secondary


to infection. This may occur as a result of a viral infection such as mumps, or bacterial
infection due to a urinary tract infection (UTI) or sexually transmitted disease,
eg Chlamydia or gonorrhoea. The epididymis is often swollen and tender.
Meckel’s diverticulum
Meckel’s diverticulum will result in an acute abdomen, however this patient’s abdomen is
soft and there would not be tenderness of the testicle.

MRCS Part A - Sep 2019 Exam


A 70-year-old retired schoolteacher presents with nocturia and poor urinary flow. He is
diagnosed with benign prostatic hyperplasia (BPH).
Which one of the following is the most appropriate treatment for him?

Alfuzosin

Amlodipine
Goserelin
Methyldopa
Oxybutynin

Explanation
Alfuzosin

Alfuzosin is a selective Alpha-blocker that relaxes prostatic smooth muscles and is typically
used as first line medical therapy for troublesome lower urinary tract symptoms.
Amlodipine

Amlodipine is a calcium-channel blocker typically used in the treatment of hypertension


and not for troublesome lower urinary tract symptoms.
Goserelin

Goserelin is a luteinising hormone-releasing hormone (LHRH) agonist that has a role in the
treatment of prostatic cancer.
Methyldopa

Methyldopa is an anti-hypertensive medication.


Oxybutynin

Oxybutynin is an anticholinergic drug that is used in the treatment of urinary frequency


due to detrusor instability (overactive bladder).

MRCS Part A - Sep 2019 Exam


You are treating a 67-year-old man with neuropathic bladder dysfunction. The patient has
an underactive detrusor and urethra.
What is the most likely cause?

Cerebrovascular accident (CVA)

Diabetes mellitus

Multiple sclerosis

Spina bifidaVesicoureteric reflux

Explanation
Diabetes mellitus
Detrusor underactivity is defined as a contraction of reduced strength and/or duration
resulting in prolonged bladder emptying and/or failure to achieve complete bladder
emptying. Poorly controlled diabetes mellitus can lead to peripheral neuropathy. Lesions of
the sacral cord or peripheral nerves cause an underactive detrusor and urethra.

Cerebrovascular accident (CVA)

Lesions above the pons cause loss of cerebral inhibition and may produce an overactive
detrusor leading to urgency, frequency and urge incontinence. Although evidence has
shown that symptoms often improve after the first few weeks.

Multiple sclerosis

Multiple sclerosis (MS) is an inflammatory-demyelinating disease of the central nervous


system. Patients commonly experience detrusor overactivity with symptoms of frequency,
urgency and urge incontinence. Involuntary contraction of the bladder often occurs
simultaneously to varying degrees of contraction of the external urinary sphincter, with
consequent sphincter asynchrony.
Spina bifida
Spina bifida can lead to dyssynergy so that the sphincter is closed when the detrusor
contracts, creating high pressures within the bladder but low flow rates.
Vesicoureteric reflux
Vesicoureteric reflux can be a complication of a neurogenic bladder, typically with detrusor
external sphincter dyssynergia, leading to high bladder storage and voiding pressures.
When the detrusor filling pressure is high ureteral drainage into bladder deteriorates,
leading to hydronephrosis and vesicoureteric reflux. As a consequence the patient can
develop recurrent urinary tract infections and renal insufficiency.
MRCS Part A - Sep 2019 Exam
A 57-year-old man is catheterised for urinary retention with a 14 French catheter,
but insertion is difficult initially. As the Urology Registrar, you are called due to scrotal and
perineal swelling. Blood is seen at the meatus and the catheter has not drained urine over
the last 3 h. The patient is now in considerable discomfort.
What will be the most appropriate management step?

Flexible or rigid cystoscopy

Cystourethrogram

Surgical repair of the urethra on a CEPOD theatre

Video urodynamics

Suprapubic catheter under ultrasound guidance

Explanation
Suprapubic catheter under ultrasound guidance

Insertion of a suprapubic catheter under ultrasound guidance (which is the gold standard)
will treat the acute urinary retention and relieve the patient of pain and discomfort.
Secondarily, it will also divert the urine away from the urethra to give it a chance to heal
over time. In this particular example, it is clear that a significant urethral injury has
occurred, so there is little alternative to immediately drain the bladder. Traumatic catheter
insertion may rupture the posterior and anterior urethra. Rupture of the spongy (penile)
part of the urethra leaks urine into the superficial perineal fascia (of Colles), which is
continuous with the membranous fascia (of Scarpa) in the anterior abdominal wall – such
urine leakage may result in swelling of the perineum and scrotum. Blood may also be seen
at the urethral meatus. A suprapubic catheter may be used to temporarily divert urine out
of the bladder and away from the injured urethra. Investigation with a retrograde
urethrogram and repair should be considered at 6–12 weeks post injury.

Flexible or rigid cystoscopy

It is likely that the urethra has suffered trauma here resulting in a false passage. It is not
appropriate to perform a cystoscopy as it could be difficult to ascertain in the acute setting
the true lumen of the urethra. The patient is also in acute urinary retention, which needs
managing.

Cystourethrogram
This option would not be conducted in the acute setting. A retrograde urethrogram should
be done electively once the injury to the urethra has had a chance to heal and can be
assessed properly.

Surgical repair of the urethra on a CEPOD theatre

This is not appropriate in the acute setting. The patient is in acute urinary retention and
this needs to be addressed first. Urine also needs to be diverted away from the urethra to
allow it to heal over time. Repair in the acute setting will result in a high chance of
complications.

Video urodynamics

Video urodynamics is important in assessing the behaviour and function of the bladder and
urinary tract. It does not play a role in this particular case.

MRCS Part A - Sep 2019 Exam


A 54-year-old man is seen under the urologists with recurrent urinary tract infections.
Imaging for colicky right flank pain shows a staghorn renal tract calculus.
Which feature is most likely to be present in relation to this calculi?
Bacteroides organismDecreased calcium

High level of urease-producing bacteria

Low in phosphate

Radio-occult

Explanation
High level of urease-producing bacteria

Urease-producing bacteria split urea into ammonia and hydroxide. These stones are
commonly formed of magnesium ammonium phosphate (struvite) or calcium carbonate
apatite. They are radiopaque and form in the renal pelvis hence the staghorn shape.
Staghorn calculi are usually seen in the setting of recurrent urinary tract infection with
urease-producing bacteria (eg Proteus, Klebsiella, Pseudomonas and Enterobacter).
Bacteroides organism
Urease-producing bacteria such as proteus, Klebsiella, Pseudonomas and Enterobacter (not
bacteroides) increase the risk of stag horn calculi.
Decreased calcium
Staghorn calculi can be formed of struvite or calcium carbonate apatite, and therefore
increased calcium is likely to be seen.

Low in phosphate

Struvite is formed of magnesium ammonium phosphate and therefore is high in phosphate.


Radio-occult
Staghorn calculi are radiopaque and therefore can be seen on a range of imaging, including
plain film, computed tomography (CT) scan and ultrasound.

MRCS Part A - Sep 2019 Exam


A 7-year-old boy presents with fever, abdominal pain, nausea and vomiting and
haematuria. Wilms’ tumour is diagnosed.
Which one of the following statements about Wilms’ tumour is correct?
It commonly presents at between 5 and 10 years of ageIt most commonly presents with
haematuriaIt is bilateral in 40% of casesMetastases commonly involve the liver and
lungNephrectomy with adjuvant chemotherapy is the mainstay of treatment

Explanation
Nephrectomy with adjuvant chemotherapy is the mainstay of treatment
If treated early with nephrectomy and postoperative chemotherapy, it carries a favourable
prognosis of around 80% survival rate at 3 years.
It commonly presents at between 5 and 10 years of age
Wilms’ tumour or nephroblastoma is the commonest intra-abdominal malignancy in
children under 10 years. It affects about 1 in 10 000 live births and commonly presents
between 0 and 3 years.
It most commonly presents with haematuria
Wilms’ tumour presents as an abdominal mass in 90% of cases, although it may also
present with haematuria (30%) and abdominal pain (20%).
It is bilateral in 40% of cases
Wilms’ tumour is bilateral in approximately 5% of cases.
Metastases commonly involve the liver and lung
Metastases in Wilms’ tumour are typically from haematogenous spread. The most common
site of spread for Wilms’ tumour is the lung. Metastatic spread can happen to the liver and
brain but this is not common.

MRCS Part A - Sep 2019 Exam


A patient has a routine urine dip. They had presented to A&E with indeterminate
abdominal pain.
Which one of the following renal functions will be assessed if you are measuring the
urine specific gravity?

Blood flow

Concentration

Filtration

Reabsorption

Secretion

Explanation
Concentration

Renal concentrating ability is reflected by specific gravity. In humans, normal specific


gravity values range from 1.002 g/ml to 1.028 g/ml. Increased specific gravity (ie increased
concentration of solutes in the urine) is associated with dehydration, glucosuria and the
syndrome of inappropriate antidiuretic hormone secretion (SIADH). Decreased specific
gravity (ie decreased concentration of solutes in urine) is associated with renal failure,
pyelonephritis, diabetes insipidus, acute tubular necrosis, interstitial nephritis and
excessive fluid intake. Paired osmolality is more commonly used to assess renal
concentrating ability.

Blood flow

Urine specific gravity refers to the concentration of solutes in the urine. Accurate
measurement of a patient’s renal blood flow is challenging with renal function used as a
proxy for adequate renal blood flow.

Filtration

Filtration is often measured clinically through the estimated glomerular filtration rate
(eGFR).

Reabsorption

Reabsorption that occurs in the kidney is often measured through proxy measurements of
renal function.

Secretion
Urine specific gravity is a measure of renal concentrating ability.

Contact Us

• MRCS Part A - Sep 2019 Exam


A 22-year-old woman with previous renal colic presents with severe left loin pain radiating
to her groin. She reports rigors and on examination looks unwell with a temperature
of 38.5°C. Urine dip shows blood 3+ nitrites and leukocytes. A CT scan shows a 7 mm
stone in the left pelviureteric junction with proximal hydronephrosis.
What would be the most appropriate management for this?
Catheterisation with an irrigation catheterFlexible cystoscopy
Insertion of a nephrostomyLaparoscopy
Open surgery

Explanation
Insertion of a nephrostomy
This case demonstrates an obstructed and infected urinary tract and as such constitutes a
urological emergency with a danger of pyonephrosis developing unless the obstruction is
urgently relieved. A nephrostomy tube can be placed percutaneously to allow drainage of
the kidney. In addition to this, other measures include intravenous (IV) antibiotics and fluid
resuscitation. A ureteric stent would be another option.
Catheterisation with an irrigation catheter
This treatment can be used when there is obstruction of the bladder, however, the CT scan
demonstrates that the stone is in the left ureter.
Flexible cystoscopy

Flexible cystoscopy allows visualisation of the bladder, but not the ureters.
Laparoscopy

Laparoscopy is not indicated at present, instead a percutaneous nephrostomy tube should


be placed.
Open surgery
Open surgery carries significant risk and recovery times and is not required in this setting.
Placement of the nephrostomy tube and treatment with IV fluids and antibiotics is
sufficient at this stage.

MRCS Part A - Sep 2019 Exam


A 38-year-old man reports sudden-onset, severe, right flank pain that comes in waves. His
urine dipstick shows no ketones, glucose, protein, nitrite or urobilinogen; the urine
contains blood, and a few white blood cells. The specific gravity of the urine is 1.015
(normal value 1.002–1.030) and the pH is 5.5 (normal value 6).
Which one of the following is the most likely diagnosis?

Benign prostatic hyperplasia

Membranous glomerulonephritis

Renal angiomyolipoma

Transitional cell carcinoma of bladder

Ureteric calculus

Explanation
Ureteric calculus

This is a classic history of ureteric colic. During passage through the ureter a calculus
irritates the ureter and can become lodged, obstructing urine flow and causing hydroureter
and hydronephrosis. There are three anatomical narrowings in the ureter, where stones
are most likely to become lodged; the ureteropelvic junction, the ureteral crossing of the
iliac vessels and the ureterovesical junction. Distal stones and stones < 5 mm diameter are
more likely to pass spontaneously. Even partial obstruction causes decreased glomerular
filtration, which can persist briefly after the calculus has passed. With hydronephrosis and
elevated glomerular pressure, renal blood flow declines, this further worsens renal
function. Generally, however, permanent renal dysfunction occurs only after about 28 days
of complete obstruction. Secondary infection can occur with longstanding obstruction. The
three main indications for hospital admission with ureteric stones are:

• fever – any evidence infection in the presence of obstruction will usually require
hospital admission and surgical intervention to relieve obstruction (nephrostomy,
retrograde ureteric stent, antegrade ureteric stent)
• impaired renal function
• pain requiring IV analgesia
• single kidney – post-nephrectomy or non-functioning contralateral kidney.

Benign prostatic hyperplasia


Benign prostatic hypertrophy is seen in men of advancing age in which they present with
lower urinary tract symptoms such as poor flow, hesitancy, and dribbling. Urinalysis may
reveal microscopic haematuria.

Membranous glomerulonephritis

Membranous glomerulonephritis will not present flank pain.

Renal angiomyolipoma

This is a renal parenchymal tumour, which is often asymptomatic and is more common in
women.

Transitional cell carcinoma of bladder

Transitional cell carcinoma of the bladder would be uncommon in a man of this age and
would not present with renal colic, instead it presents with haematuria (either frank or
microscopic) usually above the age of 60, more commonly in men. The main two risk
factors are smoking and occupational exposures (rubber, leather dye, paint, printing and
aromatic amines).

• MRCS Part A - Sep 2019 Exam


You are on an intensive care unit (ITU) ward round where you review a patient admitted
with severe sepsis and multiorgan failure. As a result he has low urine output and
peripheral oedema. He is acidotic and hyperkalaemic.
Which one of the following is correct regarding renal replacement therapy?
It can only be delivered via a central lineIt is a cure for kidney diseaseIt may involve
replacing filtered fluid in the peritoneal cavityIt should be considered first line for fluid
overloadRapid ultrafiltration of the blood cannot be undertaken

Explanation
It may involve replacing filtered fluid in the peritoneal cavity
Renal replacement therapy is indicated in intractable fluid overload (ITU), hyperkalaemia,
acidosis and uraemia when medical therapy has failed. It can be administered as a holding
measure to cover an acute crisis or in the chronic setting. In the acute setting, such as on
ITU, haemofiltration is usually used, in which the patient’s own blood pressure drives the
formation of an ultrafiltrate. The advantage over haemodialysis is that the machinery
involved is less complex and it can continue for many hours. In the chronic setting, in which
the maintenance of a reasonable lifestyle of the patient is requisite and the process must be
rapid/intermittent, haemodialysis or peritoneal dialysis is commonly used. Haemodialysis
like haemofiltration involves the patient’s blood being pumped through an artificial kidney
where waste products are exchanged with dialysis fluid, which surrounds the semi-
permeable tubes through which the blood is passed. Often an arteriovenous fistula is
formed surgically to allow easy vascular access for haemodialysis. Peritoneal dialysis
involves fluid being pumped into the abdominal cavity into which waste products diffuse.
The fluid must be changed episodically (often four times per day).
It can only be delivered via a central line
Renal replacement therapy can be delivered via a central line, tunnelled lines,
arteriovenous fistulae or through intraperitoneal catheters.
It is a cure for kidney disease
Renal replacement therapy can be used to extend a patient’s life in the setting of chronic
renal failure, or as a temporary measure during severe acute kidney injury, it is not
however a cure for kidney disease.
It should be considered first line for fluid overload
Renal replacement therapy (RRT) should only be considered for intractable fluid overload.
Conservative and medical therapy, such as fluid restriction and diuretics, should be
attempted before RRT. Other indications for renal replacement therapy in the acute setting
include: severe acidosis (<7.1), intractable hyperkalaemia, certain drug overdoses, and
signs of uraemia (eg pericarditis or encephalopathy).
Rapid ultrafiltration of the blood cannot be undertaken
Rapid ultrafiltration can be undertaken with renal replacement therapy through
haemodialysis.
1796

• MRCS Part A - Sep 2019 Exam


A 55-year-old woman attends the Urology Clinic complaining of a 2-week history of visible
haematuria and increased urinary frequency as well as occasional flank pain. She has
recently undergone an elective rigid cystoscopy and intradetrusor Botox injections. She has
not been sexually active in the last year.
What is the most likely pathogen responsible for her urinary tract infections (UTI)?
E. coliStreptococcus faecalisPseudomonas spp.

Staphylococcus saprophyticus

Staphylococcus aureus

Explanation
Staphylococcus aureus
Staphylococcus aureus is the most likely pathogen given a history of recent surgery on the
urinary tract.
E. coli
E. coli is responsible for most UTIs in the community however this patient reports recent
instrumentalisation which makes Staphylococcus aureus more likely to be the organism
responsible. UTIs are the most common bacterial infections in all age groups. Women are at
greater risk of UTIs than men because a shorter urethra allows transfer of faecal flora;
transfer of vaginal flora occurs during sexual intercourse and hormonal changes cause
changes in urine composition during pregnancy and the menopause.
Streptococcus faecalis
Streptococcus faecalis (Enterobacteria) is a less common cause of acute uncomplicated UTI
in both sexes and at all ages.
Pseudomonas spp.
Pseudomonas is usually indicative of a foreign body ie: mesh, suture.

Staphylococcus saprophyticus

Staphylococcus saprophyticus is particularly seen in sexually active women and the


question states that she has not been sexually active in the last year.

MRCS Part A - Sep 2019 Exam


You are checking a patient's results from a 99Tcm diethylenetriamine penta-acetic acid
(DTPA)-renogram. The report states that there is a hypervascular flush.
What condition is the patient most likely to have?
A functioning renal transplantObstructive uropathySimple renal cystsSpace-occupying
lesion of the kidneyVesicoureteric reflux

Explanation
Space-occupying lesion of the kidney
Increased flow within an area of the kidney (hypervascular flush) is indicative of a renal
tumour.
A functioning renal transplant
DTPA renograms are used to assess the overall function of a transplanted kidney. They are
important in the early identification and management of renal transplant complications.
Obstructive uropathy
With obstructive uropathy the DTPA renogram will not demonstrate drainage of the
affected kidney.
Simple renal cysts
Simple renal cysts appear as ‘cold spots’ on the DTPA renogram as they have no blood
supply.
Vesicoureteric reflux
DTPA renograms can determine the presence or absence of vesicoureteric reflux while
assessing the function of the kidney.
1416
Rate this question:

• MRCS Part A - Sep 2019 Exam


A 20-year-old man presents with a painless lump in his testicle. He undergoes an
ultrasound which is suspicious for malignancy.
Which one of the following statements is true of testicular malignancies?

Testicular tumours can present with gynaecomastia

Teratomas are the commonest type of testicular malignancy

In pure seminomas a-fetoprotein (AFP) is markedly raised

Teratomas are less aggressive than seminomas

An orchidectomy for malignancy should be performed through a scrotal incision

Explanation
Testicular tumours can present with gynaecomastia

Gynaecomastia is a rare presenting feature but can occur in sex hormone-secreting


interstitial-cell tumours, and may be noticed before the lump is seen.

Teratomas are the commonest type of testicular malignancy

Seminomas are germ-cell tumours arising from the seminiferous epithelium and are the
commonest form of malignant testicular tumour, accounting for 40% of tumours.

In pure seminomas a-fetoprotein (AFP) is markedly raised

AFP is not produced by pure seminomas, it is produced by yolk-sac elements, which are not
present in seminomas. b-Human chorionic gonadotrophin (b-hCG) and AFP are used to aid
diagnosis and monitoring of response to treatment in teratomas. In seminomas b-hCG is
more useful as a tumour marker, along with lactate dehydrogenase, which is less specific
but can give an idea of tumour burden.

Teratomas are less aggressive than seminomas

Teratomas are more aggressive than seminomas. Seminomas tend to spread by the
lymphatics to the para-aortic nodes, whereas teratomas spread haematogenously and
occasionally to lymph nodes.

An orchidectomy for malignancy should be performed through a scrotal incision

Any orchidectomy performed for malignancy must be performed through an inguinal


incision, not a scrotal one. This is done to allow control of the blood vessels and the vas.
These vessels must both be divided above a tourniquet placed on them. If the scrotum is
incised there is a risk of seeding and recurrence. If there is doubt, a frozen section can be
done but should also be through an inguinal incision.

MRCS Part A - Sep 2019 Exam


A 64-year-old man presents to his GP with problems passing urine. He reports hesitancy
and terminal dribbling; further questioning reveals he has been getting up several times at
night to pass urine. An ultrasound guided prostate biopsy reveals localised prostate cancer
and the patient is planned to have radiotherapy.
Which one of the following is an appropriate symptom relieving treatment option?
DutasterideFinasteridePelvic exenterationTamsulosinTransurethral resection of the
prostate (TURP)

Explanation
Transurethral resection of the prostate (TURP)
In patients undergoing radiotherapy with significant lower urinary tract symptoms there is
a risk the patient will go into retention and they require definitive treatment before
radiotherapy. A TURP would be sufficient in this case.
Dutasteride
Dutasteride is a 5α-reductase inhibitor; 5α-reductase converts testosterone to its more
active form dihydrotestosterone, which is involved in prostatic growth. Over time with
drugs such as dutasteride the prostate can reduce in size and improve lower urinary tract
symptoms but it would not be the first-line choice in this case.
Finasteride
Finasteride is a 5α-reductase inhibitor that can reduce the size of the prostate and improve
lower urinary tract symptoms. It is often used in combination with Tamsulosin for patients
with large prostates that are trialling medical therapy first. It would not be the most
appropriate treatment for this patient who is about to undergo radiotherapy.
Pelvic exenteration
Pelvic exenteration is radical surgery removing all organs from the pelvis used to treat
cancer. This patient is undergoing radiotherapy for the treatment of their prostate cancer.
Tamsulosin
Tamsulosin is an α-blocker causing a relaxation of the smooth muscle in the bladder neck
and prostate proven to improve lower urinary tract symptoms. It is typically used as first
line therapy for patients with troublesome lower urinary tract symptoms when surgery is
not indicated but would not be suitable in this patient about to undergo radiotherapy.
1431

MRCS Part A - Sep 2019 Exam


A 69-year-old smoker presents with painful haematuria and weight loss. He is diagnosed
with squamous-cell carcinoma of the bladder.
Which one of the following is correct regarding this diagnosis?
It is associated with bladder exstrophyIt accounts for 25% of all bladder cancers in the UKIt
can occur in patients with long-term indwelling cathetersIt is associated with acute
infectionIt is uncommon in areas where schistosomiasis is endemic

Explanation
It can occur in patients with long-term indwelling catheters
A risk factor for the development of squamous-cell carcinoma is chronic inflammation and
irritation, which can be caused by long-term catheters.
It is associated with bladder exstrophy
Bladder exstrophy is a congenital abnormality when the bladder is open and exposed on
the outside of the abdomen. It is most commonly associated with adenocarcinoma of the
bladder.
It accounts for 25% of all bladder cancers in the UK
Transitional-cell carcinoma is the most common type of bladder carcinoma. Squamous-
cellcarcinoma accounts for 3–7% of cases.
It is associated with acute infection
Squamous-cell carcinoma is not associated with acute infection although symptoms of
acute infection such as dysuria and frequency without evidence of infection on urinalysis
can be an indication of malignancy.
It is uncommon in areas where schistosomiasis is endemic
Schistosomiasis is a risk factor for squamous-cell bladder cancer and in regions
where schistosomiasis is endemic squamous-cell carcinoma can account for up to 70% of
bladder cancers.
1397

• MRCS Part A - Sep 2019 Exam


A 68-year-old man who had been feeling unwell and pyrexial for 10 days was seen in the
Emergency Department. He described dull pain on palpation of his left lower back. He had
burning dysuria. A full blood count revealed an elevated white blood cell count with a left
shift.
Which one of the following urinalysis findings would be most diagnostic for his renal
condition?

Hyaline casts

Oval fat bodies

Proteinuria

Renal tubular epithelial cells

White blood cell casts

Explanation
White blood cell casts

White blood cell casts indicate an inflammatory process within the kidney. This includes
glomerulonephritis and tubulointerstitial nephritis. The most common causes of
tubulointerstitial nephritis are; allergic reactions, toxins/medication reactions (NSAIDS,
phenytoin, penicillins and more), infection (pyelonephritis). Tubulointerstitial nephritis is
a primary injury to the renal tubules and interstitium that results in decreased renal
function.

Hyaline casts

Hyaline casts (formed from solidified mucoprotein) are the most common casts found in
the urine on microscopy and can be seen in healthy patients. Increased numbers may occur
as a result of chronic renal disease, strenuous exercise, and diuretic use.

Oval fat bodies

This refers to the presence of lipids in the urine and is most commonly seen as a result of
nephrotic syndrome. Burning dysuria, back pain and pyrexia would not be seen with
nephrotic syndrome.

Proteinuria

Proteinuria is non-specific and could be as a result of many different conditions, such as


nephrotic syndromes, glomerular disease, and autoimmune nephropathies.
Renal tubular epithelial cells

These are seen often in viral renal disease (such as cytomegalovirus (CMV) nephritis) and
kidney transplant rejection.
1461

• MRCS Part A - Sep 2019 Exam


A 36-year-old man is being investigated for subfertility and, on further questioning, reports
a dragging sensation in his scrotum that is exacerbated on standing. On examination the
right testicle feels normal, but on the left distended blood vessels are seen that feel like
a bag of worms. However, on lying down, they disappear.
What is the most likely diagnosis?
Hydrocoele
Indirect inguinal hernia
Orchitis
Testicular tumour
Varicocele

Explanation
Varicocele
A varicocele may be secondary to an incompetence of the valves between the left testicular
vein and left renal vein causing dilated veins of the pampiniform plexus. These often
present as a dull ache in the scrotum and feel like a ‘bag of worms’ in the scrotum. They
become more obvious when the patient stands up. They can be treated if they are
symptomatic or in cases in which patients have reduced fertility (although this does not
seem to affect pregnancy rates) by surgery or embolisation.
Hydrocoele

In a hydrocoele the testicle is often impalpable and it is usually painless.


Indirect inguinal hernia

An inguinal hernia extending into the scrotum would cause a lump and it is not possible to
‘get above it’.
Orchitis

Orchitis refers to inflammation of the testicle, most commonly seen in the context of
epididymo-orchitis. Epididymo-orchitis is an inflammatory condition of the testicle and
epididymis secondary to infection. This may occur as a result of a viral infection such as
mumps, or bacterial infection due to a urinary tract infection (UTI) or a sexually
transmitted disease eg Chlamydia infection or gonorrhoea. The epididymis is often swollen
and tender.
Testicular tumour

Testicular tumours often present as painless, palpable lumps often found on self-
examination.


• MRCS Part A - Sep 2019 Exam

You are treating a 52-year-old man with renal calculi. He has already had three admissions
to A&E with renal colic.
What can predispose individuals to renal tract calculi?

Hyperthyroidism

GoutHypoparathyroidismHypothyroidismOsteomalacia

Explanation
Gout
Stones in the renal tract occur due to an increased concentration of urinary solutes such as
calcium, uric acid and oxalic acid, or due to an increased concentration of urine during
dehydration. Gout is caused by high uric acid levels and patients with gout are also at an
increased risk of forming uric acid stones as uric acid deposits build in the kidney.

Hyperthyroidism

Thyroid dysfunction does not typically encourage renal stone formation.


Hypoparathyroidism
Hyperparathyroidism not hypoparathyroidism is linked with renal calculi formation due to
increased serum calcium and subsequent hypercalciuria, which lead to the formation of
renal calculi.
Hypothyroidism
Hypothyroidism has no proven association with renal calculi although it is associated with
reduced glomerular filtration rate.
Osteomalacia
Osteomalacia reduces serum calcium and is therefore not linked to renal calculi formation..
1414

• MRCS Part A - Sep 2019 Exam


A 35-year-old man presented in the surgical outpatient clinic complaining of a left scrotal
swelling. On examination, he had a painless, transilluminant swelling at the upper pole of
the left testis. No cough impulse was visible or palpable.
What is the most likely diagnosis?

Haematocele

Inguinal hernia

Pyocoele

Spermatocele

Varicocele

Explanation
Spermatocele

A spermatocele is a painless fluid-filled cyst that usually occurs at the upper pole of the
testis, adjacent to the epididymis and appears as a cystic scrotal mass. A large spermatocele
might be difficult to differentiate from a hydrocoele, which is also cystic, painless and
transilluminates. Ultrasonography can be helpful in reaching a diagnosis. Surgical excision
is indicated if the spermatocele becomes large and symptomatic.

Haematocele

Testicular haematocele are usually a result of trauma and occur when the bleeding remains
within the tunica vaginalis, if the tunica vaginalis is broken then a scrotal haematoma
would form. On examination there is typically a swelling with overlying ecchymosis.

Inguinal hernia

It is possible to differentiate between a scrotal swelling and inguinal hernia on examination


alone. If the examiner can get above the swelling then it is arising from the scrotum, if not
then it is more likely to be an inguinal hernia. In addition hernias when not incarcerated
would have a cough impulse.

Pyocoele

Pyocoeles are purulent fluid collections typically in the scrotal sac that can occur as a
complication of epididymo-orchitis. It is an uncommon presentation but must be
recognised and quickly treated as it can lead to Fournier’s gangrene. They typically present
with a short history of scrotal swelling, pain and erythema.
Varicocele

A varicocele is an enlargement of the pampiniform venous plexus in the scrotum. On


examination the classical description is that the scrotal sac feels like a bag of worms. Do not
forget that a left-sided varicocele could be the only presenting symptom of a left renal
cancer, as the left gonadal vein drains into the left renal vein.

MRCS Part A - Sep 2019 Exam


You are examining a 40-year-old man patient who presents with a painless scrotal lump
which he has now had for 5 years and is increasing in size. On examination, it is non-
tender, soft and not felt distinct from the testis.
What is the most likely diagnosis?
Epidydimal cyst
Gumma
Hydrocoele
Teratoma
Varicocoele

Explanation
Hydrocoele

This is the most likely diagnosis as it fits the criteria of a hydrocele:

• painless
• difficult or impossible to feel the testis, which is surrounded by a fluid collection
• often chronic
• slowly increases in size over time.

Epidydimal cyst

Epididymal cysts are often felt distinctly from the testis itself and are often multiple.
Gumma

Gummas are rare and are due to syphilis of the testis, resulting in a round, hard insensitive
mass involving the testis which is difficult to distinguish from a tumour.
Teratoma

Usually a solid mass, unlikely to be the most likely diagnosis as it would have disastrous
consequences if present for 5 years.
Varicocoele
It is arguable that this is a possible diagnosis, however not the most likely one. It does not
feel cystic in nature and, conversely, is often described as feeling ‘like a bag of worms’ and
can be responsible for a dragging sensation or an ache in the scrotum. This condition is
often painful.

• MRCS Part A - Sep 2019 Exam


An elderly man presents with painless haematuria. The last episode in the past was painful.
Which one of the following statements is true about his possible diagnosis?
Painless haematuria cannot be a urinary calculus

Squamous-cell carcinoma is associated with exposure to certain dyes, and smokers

Squamous-cell carcinomas are usually solid and invasive

Carcinoma in situ is not a precursor to invasive carcinoma

Metastatic lung carcinoma does not produce bladder secondaries

Explanation
Squamous-cell carcinomas are usually solid and invasive

Painless haematuria is the most common presentation of a bladder cancer and initial
investigations may include imaging such as an ultrasound scan or a computed tomography
(CT) urogram in addition to a flexible cystoscopy. At flexible cystoscopy, the squamous-
cellcarcinoma appears as a solid lesion commonly found on the trigone or lateral walls. It is
typically very invasive and therefore often high grade at diagnosis.
Painless haematuria cannot be a urinary calculus
Urinary calculi can cause painless haematuria and is one of the considerations when
investigating a patient in the elective setting, although a painful haematuria is more
commonly seen in the hospital setting, for example in renal colic.

Squamous-cell carcinoma is associated with exposure to certain dyes, and smokers

Squamous-cell carcinoma arises from metaplastic squamous epithelium and is associated


with chronic irritation of the bladder for example from long-term catheter use or in parts of
the world where the parasitic infection schistosomiasis is more common. Transitional-
cellcarcinoma is associated with some aniline dyes in the textile industry, reagents in
rubber, smoking and analgesic abuse.

Carcinoma in situ is not a precursor to invasive carcinoma

Carcinoma in situ is a type of non-muscle-invasive bladder cancer. It appears as flat red


lesions within the bladder. It is high grade, meaning that it is more likely to recur and
progress than other non-muscle-invasive bladder cancers.
Metastatic lung carcinoma does not produce bladder secondaries
Secondary tumours, by haematogenous and lymphatic spread, can be found in the
bladder,although they are rare. Direct spread from the cervix, prostate or rectum is more
common.
1368

• MRCS Part A - Sep 2019 Exam


A 14-year-old man presents with a 1 h history of severe testicular pain and is rushed to
theatre for an emergency scrotal exploration. The testis appears to be torted and
is repairedin the usual fashion. At closure, the surgeon is satisfied that normal perfusion
has been restored.
The patient has an increased risk of which condition?
Chronic groin pain
Increased risk of cancer in the contralateral testis
Inguino-scrotal hernia

Epididymal cyst

Varicocele

Explanation
Increased risk of cancer in the contralateral testis

Evidence exists to indicate that, following testicular torsion, there is an increased risk of
testicular cancer in both the ipsilateral and contralateral testis. The reason for this is
unclear.
Chronic groin pain

Chronic groin pain is not a typical complication of testicular torsion or scrotal exploration
but can happen following inguinal hernia repair. Chronic scrotal pain is associated with
testicular torsion and scrotal exploration.
Inguino-scrotal hernia

There is no link between testicular torsion or scrotal exploration and inguino-scrotal


hernia. However, there is a link with open appendicectomy and inguinal hernias thought to
be due to damage to the nerve supply during the incision resulting in weakness of the deep
inguinal ring.

Epididymal cyst

Epididymal cysts are fluid-filled cysts arising from the epididymis. They are a common
finding and do not require treatment unless symptomatic. The reason for their
development is unclear but there is no known link with testicular torsion.

Varicocele

There is no reported risk of varicocele after testicular torsion. Varicocele can cause
discomfort and may be a differential to consider in a patient presenting with testicular
pain,but should be easy to distinguish from a torsion on examination with its classical
description of a bag of worms.


• MRCS Part A - Sep 2019 Exam

A 21-year-old man takes advantage of his employer’s health benefits and undergoes a
private health screen. Urinalysis is positive for protein. His serum urea and creatinine
concentrations are normal.
Which one of the following options would be the most important investigations to
perform next?

History, blood pressure, protein:creatinine ratio (PCR) and fasting blood glucose

History, blood pressure, albumin: creatinine ratio (ACR), fasting blood glucose and urine
protein electrophoresis

History, blood pressure, mid-stream urine for culture and sensitivity and fasting blood
glucose

History, blood pressure, 24-h urine collection and urine protein electrophoresis

History, blood pressure and 24-hour urine collection

Explanation
History, blood pressure, albumin: creatinine ratio (ACR), fasting blood glucose and urine
protein electrophoresis

This is the correct answer; a thorough work-up is important to ascertain the aetiology of
proteinuria. Its presence generally suggests renal disease or multiple myeloma.

History, blood pressure, protein:creatinine ratio (PCR) and fasting blood glucose

To detect and identify proteinuria, use urine albumin/creatinine ratio (ACR) in preference
to PCR, because it has greater sensitivity than PCR for low levels of proteinuria.

History, blood pressure, mid-stream urine for culture and sensitivity and fasting blood
glucose

Proteinuria is not an indication of a urine infection, but can be present concurrently. It is


important to rule out an infection but is not one of the most important first line
investigations.

History, blood pressure, 24-h urine collection and urine protein electrophoresis
The main reason for performing urine protein electrophoresis is to find a light chain
myeloma producing an excess of free light chains (Bence Jones protein), an important part
of a myeloma screen. In this case, other investigations such as ACR should be done first.

History, blood pressure and 24-hour urine collection

Albumin:creatinine ratios (ACR) are generally more useful than 24-hour collections hence
are preferable. These collections are useful in metabolic screening for kidney stones.
Proteinuria can occur because of glomerular damage, tubular damage (hence the decreased
absorption of the small amount of low molecular weight proteins that are normally filtered
by the glomeruli), an increased concentration of low-molecular-weight proteins in the
blood (eg immunoglobulin light chains in myeloma – Bence Jones – protein) and secretion
of proteins into the urine (eg in urinary tract infection). Tubular proteins and Bence Jones
protein can be detected by electrophoresis of concentrated urine. However, particularly in
a young person, orthostatic proteinuria should be excluded before proceeding to further
investigation.

MRCS Part A - Sep 2019 Exam


A 49-year-old man presents with haematuria, flank mass, abdominal pain, weight loss and
back pain. They are referred to the local emergency urology service.
Which one of the following investigation findings would most likely be associated
with this presentation?

Hyponatraemia

HypercalcaemiaHypocalcaemiaLymphopaenia

Polycythaemia rubra vera

Explanation
Hypercalcaemia
Hypercalcaemia is the most common paraneoplastic syndrome in renal-cell carcinoma
affecting up to 20% of patients. It is thought to be due to the secretion of parathyroid
hormone by the tumour leading to increased bone resorption and decreased renal
clearance of calcium.

Hyponatraemia

Hyponatraemia is not caused by renal-cell carcinoma although there is some evidence as


with many other medical conditions that hyponatraemia at presentation is a poor
prognostic indicator.
Hypocalcaemia
Hypocalcaemia is not seen with renal-cell carcinoma, it is hypercalcaemia that is one of the
paraneoplastic syndromes.
Lymphopaenia
Renal-cell carcinoma does not cause lymphopaenia but there is some evidence that it is an
indicator of poor prognosis for patients with papillary renal-cell carcinoma.

Polycythaemia rubra vera

Polycythaemia is one of the paraneoplastic syndromes associated with renal-cell


carcinoma, but not polycythaemia rubra vera, which is a specific myeloproliferative
disorder.

• MRCS Part A - Sep 2019 Exam


You are called to review a 74-year-old man on the orthopaedic ward who is complaining of
severe pain in the tip of his penis, following catheterisation for urinary retention after a
total hip replacement. On examination the prepuce is retracted with the glans swollen and
oedematous.
What is the most likely diagnosis?
Balanitis
Balanitis xerotica obliterans (BXO)
Paraphimosis
Phimosis
Traumatic catheterisation

Explanation
Paraphimosis

A paraphimosis occurs when the prepuce is retracted beyond the glans and cannot be
replaced. This results in the prepuce acting as a tight band and preventing venous return,
which can result in swelling and oedema of the glans. It may be caused by not returning the
retracted foreskin to its original position after urethral catheterisation.
Balanitis

Balanitis is an acute inflammation of the glans (while ‘balanoposthitis’ implies


inflammation of the glans and foreskin) often caused by a bacterial infection.
Balanitis xerotica obliterans (BXO)

BXO is a condition in which white plaques are present on the glans and prepuce making
retraction of the foreskin difficult. It is an indication for circumcision.
Phimosis

A phimosis occurs when the foreskin is tight, making retraction difficult.


Traumatic catheterisation
Traumatic catheterisation is likely to result in frank haematuria and pain.
2618

• MRCS Part A - Sep 2019 Exam


Prostate cancer is the most common cancer in men in the UK. A bulky and hard prostate
may be found on examination.
Which one of the following is true about prostate cancer?
Prostate carcinoma arises in the paraurethral tissueMalignant change may occur in the
compressed normal tissue left behind after TURP

Average survival after diagnosis of metastatic disease is 6 months

In T1 stage from the TNM classification a physician would be able to palpate a nodule on
rectal examination

Prostate-specific antigen (PSA) is used as a screening tool for prostate cancer

Explanation
Malignant change may occur in the compressed normal tissue left behind after TURP
Malignant change can happen in any prostatic tissue left behind after transurethral
resection of the prostate (TURP). If there are clinical concerns regarding prostate cancer
then having had a previous TURP should not prevent the patient undergoing further
assessment for prostate cancer.
Prostate carcinoma arises in the paraurethral tissue
Carcinoma of the prostate arises in the peripheral glands rather than the paraurethral
glands, which is why it often takes longer to become symptomatic.

Average survival after diagnosis of metastatic disease is 6 months

With asymptomatic metastatic disease the average survival is 2–3 years, this reduces to 12
months with symptomatic disease.

In T1 stage from the TNM classification a physician would be able to palpate a nodule on
rectal examination

At the T1 stage there is clinically inapparent tumour that is not palpable, usually diagnosed
either incidentally from histology of resected prostatic tissue eg post TURP; or identified by
biopsy. With a palpable abnormality of the prostate the clinical staging would be T2 or
above.

Prostate-specific antigen (PSA) is used as a screening tool for prostate cancer

PSA is not used as a screening tool for prostate cancer, it has both a high false-
positive and false-negative rate. Other conditions that increase a patient’s PSA
include: benign prostatic enlargement; prostatitis, acute urinary
retention; and catheterisation. Patients should be appropriately counselled before being
offered a PSA test as it can lead to further invasive investigations that have significant
morbidity.
1371

• Contact Us
• MRCS Part A - Sep 2019 Exam

A 72-year-old man is being consented by the Urology specialist registrar (SpR) for
a transurethral resection of the prostate (TURP). The patient is anxious about the
procedure.
Which one of the following risks should he be warned about?

Hypernatraemia

HypokalaemiaHypomagnesaemia

Renal failure

Retrograde ejaculation

Explanation
Retrograde ejaculation
Patients should be warned of retrograde ejaculation when semen flows back into the
bladder. This is usually prevented by internal muscles close to the bladder neck that are
often damaged during TURP.

Hypernatraemia

TURP syndrome is reported as having a 0.5–2% risk in patients undergoing traditional


monopolar TURP with glycine irrigation. It is caused by a combination of dilutional
hyponatraemia, fluid overload and glycine toxicity.
Hypokalaemia
The main electrolyte risk with TURP is hyponatraemia due to TURP syndrome.
Hypomagnesaemia

The patient should be warned specifically about TURP syndrome, which classically causes
dilutional hyponatraemia.

Renal failure

TURP is not associated with a risk of renal failure and does not form part of the consent.
1433
• MRCS Part A - Sep 2019 Exam

A 54-year-old with end stage renal failure (ESRF) presents 2 days post dialysis with chills,
fever and nausea. You refer them onto the nephrologists.
Which one of the following statements regarding renal replacement therapy is
correct?
May involve replacing filtered fluid in the peritoneal cavity – so-called haemofiltration
May be indicated first line for fluid overloadMay be indicated in hypokalaemiaMay require
the formation of an arteriovenous fistulaShould not be considered in cases where rapid
ultrafiltration is required

Explanation
May require the formation of an arteriovenous fistula
In the chronic setting, where the maintenance of a reasonable lifestyle of the patient is
requisite and the process must be rapid/intermittent, haemodialysis or peritoneal dialysis
is commonly used. Haemodialysis, like haemofiltration, involves the patient’s blood being
pumped through an artificial kidney where waste products are exchanged with dialysis
fluid, which surrounds the semi-permeable tubes through which the blood is passed. Often
an arteriovenous fistula is formed surgically to allow easy vascular access for
haemodialysis.
May involve replacing filtered fluid in the peritoneal cavity – so-called haemofiltration

Renal replacement therapy can include installation of dialysis fluid into the peritoneal
cavity, utilising the peritoneal lining as a filtration membrane. This is called peritoneal
dialysis, not haemofiltration.
May be indicated first line for fluid overload
Medical therapy should be trialled before initiation of renal replacement therapy. This can
include fluid restriction and diuretics.
May be indicated in hypokalaemia
Renal replacement therapy can be used to treat intractable hyperkalaemia, not
hypokalaemia.
Should not be considered in cases where rapid ultrafiltration is required
Rapid ultrafiltration can be achieved through either haemodialysis or haemofiltration.

MRCS Part A - Sep 2019 Exam


A 2-year-old child attends the Paediatric Urology Clinic because the parents are worried
that one of his testes is missing. They have not been able to palpate the right testicle since
birth. On examination, the testis is absent from the right hemiscrotum. It is palpable within
the inguinal canal; however it cannot be milked into the scrotum.
Which one of the following is true about undescended testes (UDT)?

The incidence of undescended testicles at 1 year of age is 3%

Undescended testicles can be categorised into three types: retractile, ectopic and atrophic

High birthweight is a risk factor for developing undescended testes

The relative risk of testicular cancer is eight-fold higher. Reduced fertility is also a long-
term complication.

Inguinal undescended testes is treated with orchidectomy between 6–18 months of age

Explanation
The relative risk of testicular cancer is eight-fold higher. Reduced fertility is also a long-
term complication.

Risk of cancer, reduced fertility and increased risk of testicular torsion or trauma are all
long-term complications of undescended testes. By the age of 2, the testis is unlikely to
descend. It is preferable to perform an orchidopexy at this stage before the child becomes
of school age. Up to one-third of patients with a unilateral undescended testis have a
contralateral patent processus vaginalis (and therefore increased risk of contralateral
cryptorchidism), contralateral orchidopexy is therefore also performed.

The incidence of undescended testicles at 1 year of age is 3%

At birth (of a full-term neonate), the incidence of undescended testes is 4%, however many
testicles will spontaneously descend and at 1 year of age the incidence drops to 1.3–1.8%.

Undescended testicles can be categorised into three types: retractile, ectopic and atrophic

There are five common classifications for undescended testes:

• retractile: intermittently active cremasteric reflex results in a testis that retracts up


and out of the scrotum
• ectopic: abnormal migration of the testis below the external ring (ie perineum, base of
penis, femoral triangle)
• incomplete descent: intra-abdominal, intra-inguinal or pre-scrotal
• atrophic (absent)
• acquired UDT: testes that have ascended.

High birthweight is a risk factor for developing undescended testes

Risk factors for undescended testes include:

• family history
• twins
• pre-term infants
• low birthweight.

High birthweight is not a recognised risk factor and, in fact, infants with a high birthweight
would be less likely to have undescended testes compared with their low birthweight
counterparts.

Inguinal undescended testes is treated with orchidectomy between 6–18 months of age

Orchidopexy is the treatment of choice for undescended testes, and describes fixing of the
testes within the scrotum. Orchidectomy is removal of the testis, indicated in testicular
malignancy and occasionally when testicular trauma has occurred, most commonly in road
traffic accidents.
168


• MRCS Part A - Sep 2019 Exam

You are reviewing a patient in the urology clinic with a malignant tumour. They are due to
be discussed at the urology MDT.
Regarding urological malignancy which one of the following statements is correct?
Prostatic carcinoma commonly metastasizes to boneProstatic tumours are most commonly
squamous cell carcinomasRenal cancers are most commonly bilateralRenal cancers spread
to the lungs via the thoracic ductTesticular tumours usually spread to inguinal lymph nodes

Explanation
Prostatic carcinoma commonly metastasizes to bone
Prostatic carcinoma characteristically produces metastases in the lumbar vertebrae as a
result of the prostatic venous plexus drainage into the internal vertebral plexus.
Prostatic tumours are most commonly squamous cell carcinomas
Prostatic tumours are adenocarcinomas with a variable degree of differentiation. Prostate
cancer is graded using the Gleason Score – where tissue is graded between 0–5, the two
most abundant types of tissue are added together to form a score out of 10. Higher Gleason
scores reflect more aggressive local and metastatic spread.
Renal cancers are most commonly bilateral
Renal cell carcinomas may occasionally occur bilaterally, but they are most commonly
unilateral.
Renal cancers spread to the lungs via the thoracic duct
Renal cancer often infiltrates the adrenal gland, more distal spread is usually
haematogenous not via the lymphatic system.
Testicular tumours usually spread to inguinal lymph nodes
Testicular tumours mostly spread to the para-aortic nodes, and not to the inguinal lymph
nodes.
1464

• MRCS Part A - Sep 2019 Exam


A 69-year-old man is diagnosed with prostate cancer as an incidental finding on digital
rectal examination following presentation with lower urinary tract symptoms. He wants to
know what the ongoing management plan will be.
Which one of the following is correct in prostatic cancer?
It may be diagnosed by a serum blood test
It is an incidental finding in 90% of routine transurethral resections of the prostate
(TURPs)

It is a rare incidental post-mortem finding

There is no screening programme in the UK

It is characterised by skeletal metastases in 70% of patients at the time of diagnosis

Explanation
There is no screening programme in the UK

There is currently no screening programme in the UK. PSA screening does not fulfil the
World Health Organization (WHO) screening criteria and European studies have suggested
that it results in high levels of over-treatment of the disease.
It may be diagnosed by a serum blood test

Prostate specific antigen (PSA) is a protein produced by prostatic cells. It is produced by


both normal and cancerous cells and is therefore not specific to cancer. While changes in
the PSA can be a marker of prostate cancer and is used in active surveillance, PSA can also
be increased in benign conditions such as benign prostatic hyperplasia and prostatitis.
It is an incidental finding in 90% of routine transurethral resections of the prostate
(TURPs)
It has been reported that up to 15% of patients undergoing transurethral resection of the
prostate will have an incidental finding of prostate cancer on histology.

It is a rare incidental post-mortem finding

Prostate cancer is a very common finding at post mortem, in approximately 80% of men
over the age of 80, although figures vary depending between studies.
It is characterised by skeletal metastases in 70% of patients at the time of diagnosis
Approximately 5% of patients have skeletal metastases at the time of diagnosis of prostate
cancer. This figure has been decreased and has been attributed to the rise in the number of
patients undergoing PSA testing to identify low-grade prostate cancer and the detection of
low-grade cancer in histology following bladder outflow obstruction surgery such as
transurethral resection of the prostate or holmium enucleation of the prostate.
1408

• MRCS Part A - Sep 2019 Exam


A 77-year-old man with HIV presents with a 6-h history of severe right loin to groin
pain and is unable to get comfortable. On examination he is clearly in pain, but afebrile with
normal observations. His urinalysis shows blood 2+, but his blood tests, plain imaging and
a computed tomography (CT) scan of the kidneys, ureters and bladder (KUB) are
all unremarkable.
What is the most likely diagnosis?
Bladder stoneCalcium oxalate stoneCysteine stone
Indinavir stone
Triple phosphate stone

Explanation
Indinavir stone

Indinavir is an antiretroviral agent used in HIV that can classically cause radiolucent
stones. Given the clear history of renal colic and known diagnosis of HIV in the absence of a
stone on imaging, an indinavir stone is the most likely diagnosis.
Bladder stone
A bladder stone would tend not to cause the classic loin to groin pain associated with renal
colic, as the stone is in the bladder and therefore not causing intermittent ureteric
obstruction.
Calcium oxalate stone
Calcium oxalate, cysteine and triple phosphate stones are radio-opaque and therefore
should be seen on imaging.
Cysteine stone

A cysteine stone would be seen on radiographic imaging.


Triple phosphate stone
A triple phosphate stone is radio-opaque and therefore visible on CT imaging.

MRCS Part A - Sep 2019 Exam


A 26-year-old man presents with a hard, non-tender lump in his left testicle. Both α-
fetoprotein (AFP) and β-human chorionic gonadotrophin (β-HCG) are raised.
What is the most likely nature of the lump?

Haematoma

AbscessTeratomaSeminomaHydrocoele

Explanation
Teratoma
Teratoma would be the most common testicular tumour in this age group and can cause
both AFP and β-HCG levels to be elevated.

Haematoma

Scrotal haematoma is usually a result of trauma and can present as a firm lump but is often
painful. It does not elevate serum tumour markers.
Abscess
A scrotal abscess would typically present as a fluctuant usually painful swelling with raised
inflammatory markers. The serum tumour markers (AFP and b-HCG) would not be
elevated as a result of an abscess alone.
Seminoma
Seminomas can, but not always, elevate the tumour markers β-HCG and LDH, however a
pure seminoma does not elevate AFP.
Hydrocoele
A hydrocele is accumulation of fluid within the tunica vaginalis. A simple hydrocele would
not be associated with raised tumour markers. But it should not be forgotten that
hydroceles can be secondary to testicular tumours and in that case could be associated
with raised tumour markers.

• MRCS Part A - Sep 2019 Exam


You send a urine specimen for analysis from one of your patients who is on warfarin, and it
is reported as showing microscopic haematuria. They otherwise appear fairly well.
What should you do?
Arrange a flexible cystoscopyArrange an abdominal X-rayArrange a 24 hour urine
collectionIgnore it, the patient is taking warfarinImmediately commence antibiotics

Explanation
Arrange a flexible cystoscopy
Flexible cystoscopy is one of the first line investigations for haematuria and should be used
to investigate this patient despite the patient being on warfarin. Depending on the
indication for warfarin and local protocol it may need to be stopped before the cystoscopy.
Arrange an abdominal X-ray
First line imaging investigation for haematuria is either with ultrasound or computed
tomography (CT) urogram.
Arrange a 24 hour urine collection
A 24 hour urine collection is not recommended to investigate microscopic haematuria.
Ignore it, the patient is taking warfarin
Microscopic haematuria should not just be attributed to the patient taking warfarin, up to
25% of these patients will have an underlying problem such as malignancy.
Immediately commence antibiotics
There is no indication from the information given that this patient should be started on
antibiotics. The patient is not symptomatic and the urine specimen has only shown
microscopic haematuria, which alone does not suggest infection.
1419

• MRCS Part A - Sep 2019 Exam


A patient is being seen in urology clinic with suspected prostate cancer. They are
concerned about their ongoing investigations.
Regarding prostatic cancer (CaP), which one of the following statements is true?

Caucasians are at greatest risk of developing prostate cancer. Current detection strategies
include the efficient use of a combination of digital rectal examination (DRE), serum
prostate-specific antigen (PSA) and transrectal ultrasound (TRUS) with prostatic biopsy.

Gleason score of 15 indicates poorly differentiated tumour

PSA is the single most useful test for screening of CaP for men of all age group

About 70% of CaP arises in the peripheral zone of the prostate gland and most are
multifocal

Transurethral resection of prostate (TURP) is the treatment of choice for localised disease

Explanation
About 70% of CaP arises in the peripheral zone of the prostate gland and most are
multifocal

About 75% of adenocarcinomas are located in the peripheral zone and most (85%) are
multifocal.

Caucasians are at greatest risk of developing prostate cancer. Current detection strategies
include the efficient use of a combination of digital rectal examination (DRE), serum
prostate-specific antigen (PSA) and transrectal ultrasound (TRUS) with prostatic biopsy.

Afro-Caribbean men are at greatest risk, followed by Caucasians. The disease is uncommon
in Asians and Oriental races, depending on migration to the West. The probability of CaP
developing in a man under the age of 40 is 1 in 10 000; for men 40 to 59 it is 1 in 103; and 1
in 8 for men 60 to 79.

Gleason score of 15 indicates poorly differentiated tumour

Gleason scoring does not go beyond 10; the range is between 2–10.

• Gleason 2–6: Well differentiated


• Gleason 7: Moderately differentiated
• Gleason 8–10: Poorly differentiated

Gleason score is obtained by adding the primary and secondary grades together.
PSA is the single most useful test for screening of CaP for men of all age group

Not all patients with an elevated PSA concentration have CaP because PSA is not cancer
specific, it is prostate-specific.

Transurethral resection of prostate (TURP) is the treatment of choice for localised disease

The treatment options for localised CaP include watchful waiting, radical prostatectomy
and radiotherapy.
1349


• MRCS Part A - Sep 2019 Exam

A 75-year-old man has a serum prostate-specific antigen (PSA) concentration of 15 ng/l. He


has been referred into the urology clinic for further assessment.
Which one of the following statements is true with regard to this result?

A small proportion of poorly differentiated prostate cancers fail to express PSA

It is diagnostic of malignancy

It is likely to be invalidated if he underwent a digital rectal examination (DRE) 48 h before


the blood sample was taken

It is prognostically highly significant

It is unremarkable for a man of his age

Explanation
A small proportion of poorly differentiated prostate cancers fail to express PSA

PSA generally increases with advancing stage and tumour volume, however a small
proportion of tumours fail to express PSA.
It is diagnostic of malignancy
PSA is prostate-specific, not prostate cancer specific and hence is not, in itself, diagnostic of
malignancy.

It is likely to be invalidated if he underwent a digital rectal examination (DRE) 48 h before


the blood sample was taken

In practice, a DRE only affects the PSA by <1 ng/ml and would not invalidate the result or
account for an abnormally high result. Prostatitis and acute urinary retention can also
cause a transient rise in PSA.

It is prognostically highly significant

The absolute PSA concentration correlates poorly with prognosis in prostate cancer.

It is unremarkable for a man of his age

Over 50% of patients have extraprostatic disease if PSA >10 ng/ml hence this is a
significantly raised PSA. In general, the higher the PSA, the greater the likelihood of
malignancy, but some patients with malignancy have normal levels (often taken as less
than 4 ng/l, but are actually age-dependent).

Contact Us

• MRCS Part A - Sep 2019 Exam


A 72-year-old man presents to A&E with suprapubic discomfort and urinary retention. He
has a serum creatinine concentration of 522 mmol/l (normal 50–120 µmol/l).
Which one of the following features, if present, would help to identify possible
aetiologies for his renal failure and suggest that it is chronic?

Anaemia

Small kidneys on ultrasound examination

High serum parathyroid hormone concentration

Hypotension

Hyperuricaemia

Explanation
Small kidneys on ultrasound examination

The kidneys are usually decreased in size in chronic renal failure, due to atrophy with a loss
of nephrons and vascular tissue – except when this is due to amyloid or polycystic disease.
Kidneys are usually normal in size in advanced diabetic nephropathy, whereas affected
kidneys are initially enlarged from hyperfiltration.

Anaemia

Anaemia (due to decreased erythropoietin secretion) and secondary hyperparathyroidism


(decreased calcitriol secretion causes hypocalcaemia) are features, not causes, of chronic
renal failure.

High serum parathyroid hormone concentration

Secondary hyperparathyroidism (decreased calcitriol secretion causes hypocalcaemia) is a


feature of chronic renal failure, not a cause.

Hypotension

Hypertension, not hypotension, can be both a cause and a consequence of renal failure.
Hypotension is rarely associated with renal failure and is certainly not a cause of chronic
renal failure.

Hyperuricaemia

Uric acid nephropathy can cause renal failure but hyperuricaemia is usual in chronic renal
failure, due to decreased excretion.


• MRCS Part A - Sep 2019 Exam

A 13-year-old girl is seen in the gynaecology out-patients clinic with haematocolpos. She
noticed a mass in her abdomen.
Which of the following conditions is most likely to be associated with
haematocolpos?

Cervical condyloma

Cervical gonorrhoea

Endometriosis

Imperforate hymen

Ruptured Bartholin's cyst

Explanation
Imperforate hymen

Haematocolpos (accumulation of menstrual blood in the vagina) can cause the vagina to
bulge and in turn result in haematometra (accumulation of blood in the uterus), which can
cause uterine distension or a mass. It is most likely to be associated with an imperforate
hymen. In about 1 in 2000 women, the hymen fails to develop any opening at
all. This condition is called an imperforate hymen and if it does not spontaneously resolve
itself before puberty a physician will need to make a hole in the hymen (hymenotomy) to
allow menstrual discharge to escape.

Cervical condyloma

Cervical condylomas are related to human papillomavirus (HPV) infections and are pre-
cancerous lesions. Treatment is with excisional biopsy, diathermy or laser vaporisation.

Cervical gonorrhoea

Gonorrhoea is a sexually transmitted infection that can affect the mucous membranes of
the cervix, uterus, fallopian tubes and urethra. Untreated gonorrhoea can cause pelvic
inflammatory disease, which can lead to pelvic abscesses and chronic pelvic pain.
Endometriosis

Endometriosis is the presence of endometrial mucosa in abnormal location (other than the
uterus). It is most commonly associated with dysmenorrhoea and menorrhagia but without
another abnormality would not result in the accumulation of menstrual blood in the vagina.

Ruptured Bartholin's cyst

A Bartholin’s cyst is a small fluid-filled sac just inside the opening of the vagina, it has the
potential to become infected and if ruptured would then drain pus. It would not lead to
haematocolpos (accumulation of menstrual blood in the vagina).
1380

• Contact Us
• MRCS Part A - Sep 2019 Exam

You are examining the external genitalia of a newborn and note the presence
of hypospadias. You are concerned of the presence of a disorder of sexual development.
What is the most common location for the meatus in this condition?
Dorsal surface
Glanular
Penile
Perineal
Scrotal

Explanation
Glanular

In hypospadias the external urethral meatus is located on the ventral surface of the penis:
70% are glanular, 10% penile and 20% scrotal.
Dorsal surface

The external urethral meatus is located on the ventral, not dorsal, surface of the penis.
Penile

Only 10% of hypospadias are penile.


Perineal

Perineal placement is associated with anorectal anomalies.


Scrotal
Scrotal placement of hypospadias occurs in around 20% of cases.
3089

• MRCS Part A - Sep 2019 Exam


A 2-year-old boy presents with a palpable left-sided renal mass, associated with
a varicocele. The child is pyrexial and hypertensive. A urine dipstick is negative for blood.
What is the most likely cause?
Massive hydronephrosisNeuroblastomaRhabdomyosarcomaSimple renal cystWilms’
tumour

Explanation
Wilms’ tumour
Wilms’ tumour is the most common solid renal mass in children. The commonest
presentation is with a palpable renal mass in a pre-school age child. Haematuria is rarely
seen. Fever and hypertension may be present. Left-sided tumours can be associated with
varicoceles.
Massive hydronephrosis
Massive hydronephrosis may be diagnosed pre-nataly, approximately 50% will regress by
birth, with a further 15% resolving by age 3 years.
Neuroblastoma
Neuroblastomas cross the midline and nearly all produce catecholamines, which can be
tested for in the urine.
Rhabdomyosarcoma
Rhabdomyosarcoma is a mixed solid and cystic genitourinary malignant tumour, which
tends to present with haematuria or vaginal bleeding.
Simple renal cyst
Simple renal cysts may be an incidental finding, they are less likely to be palpable and not
usually associated with pyrexia and hypertension.
3105


• MRCS Part A - Sep 2019 Exam

A 45-year-old woman presents with right flank mass and haematuria. She also complains of
depression, constipation and vomiting.
What is the underlying diagnosis?
Colorectal carcinomaRenal stonesRenal carcinomaBladder carcinomaPyelonephritis

Explanation
Renal carcinoma
This collection of symptoms can all be explained by renal carcinoma with hypercalcaemia
as a paraneoplastic syndrome. Paraneoplastic syndromes are non-metastatic systemic
effects caused by cancer. Hypercalcaemia causes depression, abdominal pain, lethargy and
constipation.
Colorectal carcinoma
Patients with colorectal carcinoma could have a palpable abdominal mass but typically
present with a change in bowel habit.
Renal stones
Renal stones can present asymptomatically but in patients with renal colic typically present
with loin to groin pain and haematuria. These patients would not present with a mass.
Bladder carcinoma
The most common presentation of bladder cancer is haematuria occurring in 90% of
patients. These patients would not present with a flank mass and paraneoplastic
syndromes are rare.
Pyelonephritis
Pyelonephritis would be more likely to present with fever, flank pain and urinary
symptoms such as dysuria and frequency.
1456


• MRCS Part A - Sep 2019 Exam

A 78-year-old man undergoes a very prolonged transurethral resection of prostate


(TURP). He becomes unwell on the ward post-operatively.
Which one of the following options may occur?
BradycardiaHyperglycaemiaHypernatraemia
Increased osmolality of the plasma
Increased total body sodium

Explanation
Bradycardia
With prolonged TURP there is a greater risk of TURP syndrome which is in part due to fluid
overload. Fluid overload causes hypertension and reflex bradycardia.
Hyperglycaemia
Hyperglycaemia is not associated specifically with prolonged TURP.
Hypernatraemia

There is a risk of TURP syndrome with prolonged surgery. The effects are due to dilutional
hyponatraemia, fluid overload and glycine toxicity.
Increased osmolality of the plasma

With prolonged surgery there is a risk of TURP syndrome as a result of absorption of the
irrigation fluid glycine which has a lower osmolality than plasma. This can decrease the
osmolality of plasma and this puts the patient at risk of cerebral oedema, which is life
threatening.
Increased total body sodium
This is a risk of hyponatraemia with prolonged TURP as a result of TURP syndrome.
1434

• MRCS Part A - Sep 2019 Exam


A 39-year-old woman presents with fever, pain on passing urine and abdominal pain that
radiates along the flank. She is desperate for treatment and cannot cope at home.
What is the likely diagnosis?
CystitisPelvic abscess
Pyelonephritis
Urethritis

Lower urinary tract infection

Explanation
Pyelonephritis

Pyelonephritis is an ascending urinary tract infection that has ascended to affect the
kidney. Severe cases can lead to pyonephritis, urosepsis and kidney failure. Treatment is
with intravenous antibiotics and fluids.
Cystitis
Cystitis is common in women due to the short urethra and presents with dysuria. Systemic
symptoms such as fever are not seen, with pain being localised to the suprapubic region,
not the flank.
Pelvic abscess

A pelvic abscess would cause pain and fever, however, the pain would not radiate along the
flank which is suggestive of pyelonephritis.
Urethritis

Urethritis would cause dysuria only, with fever and abdominal pain unlikely.

Lower urinary tract infection

Given this patient's fever and abdominal pain, it suggests the urinary tract infection
(usually confined to the bladder) has extended proximally through the ureter and is now
affecting the kidney.
2978


• MRCS Part A - Sep 2019 Exam

A 44-year-old man presents with a testicular lump. On examination his testis feels firm and
bosselated. Scrotal ultrasonography confirms the diagnosis of a testicular tumour.
Which one of the following is true in testicular tumours?
Sertoli cell tumours always present with gynaecomastiaTeratomas are extremely
radiosensitiveThe measurement of tumour markers is useful in the management of
teratomasTeratomas are the most common tumour

Testicular tumours never present with a hydrocele

Explanation
The measurement of tumour markers is useful in the management of teratomas
Tumour markers a-fetoprotein and β-hCG are elevated in approximately 50% and 40%,
respectively, of non-seminomatous germ-cell tumours which includes teratomas. If
elevated at first presentation then they should be repeated 7 days post-orchidectomy and
are used to monitor response to treatment and recurrence.
Sertoli cell tumours always present with gynaecomastia
Sertoli cell tumours are rare and like all other testicular tumours most commonly present
as a painless lump, they can but not always present with gynaecomastia.
Teratomas are extremely radiosensitive
Primary treatment for testicular tumours usually begins with radical inguinal
orchidectomy. Post-operatively chemotherapy is often offered depending on staging.
Radiotherapy is not part of treatment for teratoma but is used for seminoma.
Teratomas are the most common tumour
Seminomas are the most common type of testicular tumours and form approximately half
of all cases.

Testicular tumours never present with a hydrocele

Patients with testicular tumours can develop secondary hydroceles in response. When
assessing a patient with a hydrocele it is important to ensure the underlying testicle is
normal with an ultrasound before surgical repair.
1406

• MRCS Part A - Sep 2019 Exam


A 52-year-old man was diagnosed with renal cell carcinoma. The tumour was seen to
be extending into the renal vein on computed tomography.
What is the clinical T stage of this tumour?

T1

T2

T3a

T3b

T3c

Explanation
T3a

These tumours can be any size but extend into the renal vein.

T1

These tumours are less than or equal to 7 cm in size that are limited to the kidney.

T2

These tumours are greater than 7 cm in size and limited to the kidney.

T3b

These tumours can be any size but extend into the inferior vena cava below the diaphragm.

T3c

These tumours can be any size but extend into the inferior vena cava above the diaphragm.
3822

You might also like